a

Menu

M

Chiudi

Test di ingresso – facoltà di Ingegneria

Test di Ingresso – Esercizi risolti e simulazioni

Home » Test di ingresso – facoltà di Ingegneria

In questo articolo raccogliamo 120 esercizi tratti dai test d’ingresso per le facoltà di Ingegneria, o quesiti a essi ispirati. Gli esercizi coprono il programma di matematica di base tradizionalmente affrontato nei vari Istituti superiori italiani e risulta pertanto un efficace banco di prova sia per chi desidera prepararsi a tali selezioni, sia per chi ricerca semplicemente un modo per testare a 360 gradi la sua preparazione in Matematica. Di ogni esercizio forniamo una soluzione dettagliata, che non si limita a riportare l’opzione corretta, ma fornisce una discussione approfondita sul metodo risolutivo, risultando così particolarmente efficace nell’apprendimento.
Buona lettura a tutti!

Consigliamo una visita alle nostre cartelle relative al programma di Matematica della Scuola superiore per il materiale teorico di riferimento e ulteriori esercizi sui medesimi temi.

Premessa

Leggi...

Questa dispensa è stata concepita con l’obiettivo di offrire una preparazione completa e rigorosa agli studenti che intendono affrontare i test di ingresso alle facoltà scientifiche, con particolare riferimento a Ingegneria, Fisica e Matematica. La raccolta include 120 esercizi selezionati, progettati per consolidare le competenze necessarie ad accedere e progredire con successo nei corsi universitari di base come Analisi 1, Geometria e Fisica 1.

 
 

Autori e revisori


 
 

Contenuti della dispensa

Leggi...

Gli esercizi proposti coprono un ampio spettro di argomenti fondamentali, corrispondenti al programma scolastico dal primo al quinto anno delle scuole superiori. Tra i principali temi affrontati si annoverano:

    \[\quad\]

  • Monomi e polinomi, incluse le tecniche di scomposizione;
  •  

  • Equazioni e disequazioni di primo e secondo grado, con estensioni al grado superiore e utilizzo dei moduli;
  •  

  • Sistemi di disequazioni;
  •  

  • Operazioni sui radicali
  •  

  • Funzioni esponenziali e logaritmiche;
  •  

  • Problemi di Geometria euclidea;
  •  

  • Geometria analitica (studio della retta, parabola, circonferenza, ellisse, iperbole);
  •  

  • Geometria solida;
  •  

  • Goniometria e trigonometria (studio delle funzioni trigonometriche, risoluzione di equazioni e disequazioni, applicazione delle formule fondamentali quali archi associati, duplicazione, somma, sottrazione e bisezione);
  •  

  • Calcolo combinatorio, statistica e probabilità.

Ogni esercizio è corredato da una spiegazione approfondita e, ove opportuno, da rappresentazioni grafiche per agevolare la comprensione dei procedimenti. L’attenzione alla chiarezza e alla precisione rende questo strumento adatto agli studenti di diversi percorsi formativi, inclusi licei scientifici, classici, linguistici e istituti tecnici.


 
 

Esercizi

 

Esercizio 1  (\bigstar\largewhitestar\largewhitestar\largewhitestar\largewhitestar). Dato r>0, la curva che nel piano Oxy ha equazione (x-1)^2+y^2=r^2 è:

    \[\quad\]

  1. una retta;
  2.  

  3. una circonferenza;
  4.  

  5. una parabola;
  6.  

  7. un’ellisse;
  8.  

  9. un’iperbole.

Svolgimento.

Risposta 2.

La distanza al quadrato fra due punti (x,y) e (x_0, y_0) nel piano è

    \[ (x-x_0)^2+(y-y_0)^2. \]

Quindi l’insieme dei punti (x,y) del piano che verificano l’equazione data nell’esercizio distano dal punto (1,0) esattamente r (vedi figura 1)

    \[\quad\]

    \[\quad\]

Rendered by QuickLaTeX.com

Figura 1: circonferenza centrata in (1,0) e di raggio r.

    \[\quad\]

    \[\quad\]

Il luogo di tali punti è quindi una circonferenza e la risposta corretta è la 2.


 

Esercizio 2  (\bigstar\bigstar\largewhitestar\largewhitestar\largewhitestar). Il dominio di esistenza di f(x)=\log_{10}\dfrac{x}{x^2-9} è:

    \[\quad\]

  1. tutto \mathbb R;
  2.  

  3. (3, +\infty);
  4.  

  5. (0, +\infty);
  6.  

  7. (-3, 0)\cup (3, +\infty);
  8.  

  9. (-\infty ,-3)\cup (0, 3).

Svolgimento.

Risposta 4.

Il logaritmo di un numero in una certa base è definito solo quando l’argomento è positivo, perciò il dominio di esistenza della funzione f è l’insieme dei punti x per cui

    \[ \frac{x}{x^2-9}>0.  \]

Dobbiamo studiare quindi i segni di x e di x^2-9=(x+3)(x-3) e selezionare solo l’insieme dei valori x per cui il prodotto dei loro segni è positivo.

Lavoriamo con l’usuale diagramma:

    \[\quad\]

    \[\quad\]

    \[\quad\]

    \[\quad\]

Quindi si ottiene

    \[ -3< x <0, \quad x>3. \]

Il grafico della funzione f è mostrato in figura 2.

    \[\quad\]

    \[\quad\]

Figura 2: grafico della funzione f.

    \[\quad\]

    \[\quad\]

La risposta giusta è quindi la 4.


 

Esercizio 3  (\bigstar\largewhitestar\largewhitestar\largewhitestar\largewhitestar). Quanti sono i termini di una successione geometrica di ragione uguale a 2 con primo termine 8 ed ultimo termine 512?

    \[\quad\]

  1. 11;
  2.  

  3. 8;
  4.  

  5. 10;
  6.  

  7. 7;
  8.  

  9. 6.

Svolgimento.

Risposta 4.

Dati una coppia di numeri a ed r, una successione geometrica di ragione r e primo termine a è una successione

    \[ a_1:=a \qquad a_2:=ar \qquad \cdots \qquad  a_n:=ar^{n-1} \qquad \cdots  \]

Una successione così fatta ha la peculiarità che il rapporto tra un termine ed il precedente è sempre costante ed è uguale alla ragione r. Nel caso dell’esericizio a=8 e r=2, dobbiamo quindi risolvere

    \[ a_n=8\cdot 2^{n-1}=512 \implies 2^{n-1}=\frac{512}{8}=64=2^6 \implies n=7. \]

La risposta corretta è quindi la 4.


 

Esercizio 4  (\bigstar\largewhitestar\largewhitestar\largewhitestar\largewhitestar). Il luogo dei punti dello spazio definito dall’equazione (x-1)^2+y^2+(z+\sqrt{2})^2=0 è:

    \[\quad\]

  1. un punto;
  2.  

  3. una sfera non degenere;
  4.  

  5. un cilindro non degenere;
  6.  

  7. vuoto;
  8.  

  9. nessuna delle precedenti.

Svolgimento.

Risposta 1.

Una somma di quadrati è sempre maggiore o uguale a zero ed è zero se e soltanto se tutti gli addendi sono uguali a 0. Otteniamo quindi

    \[ x-1=0, \qquad y=0 \qquad \makebox{e} \qquad z+\sqrt{2}=0. \]

Il luogo è quindi un solo punto, (1, 0, -\sqrt{2}).


 

Esercizio 5  (\bigstar\bigstar\largewhitestar\largewhitestar\largewhitestar). Il volume di un prisma è uguale a:

    \[\quad\]

  1. il quadrato dell’altezza per il perimetro della sua base;
  2.  

  3. il doppio dell’area della base per l’altezza;
  4.  

  5. tre volte il volume di una piramide avente la stessa base e altezza del prisma;
  6.  

  7. l’area della base per il quadrato della sua altezza;
  8.  

  9. nessuna delle precedenti.

Svolgimento.

Risposta 3.

Il volume V_p di un prisma è uguale al prodotto dell’area di base b per la sua altezza h. D’altro canto, la formula per il calcolo del volume V_{pir} di una piramide avente stessa base e altezza del prisma è

    \[ V_{pir}=\frac{b\cdot h}{3}=\frac{V_p}{3} \implies V_{p}=3\cdot V_{pir}.  \]

In figura 3 diamo un esempio di prisma e piramide a base pentagonale.

    \[\quad\]

    \[\quad\]

Figura 3: esempio di prisma e piramide a base pentagonale.

    \[\quad\]

    \[\quad\]

Perciò la risposta corretta è 3.


 

Esercizio 6  (\bigstar\largewhitestar\largewhitestar\largewhitestar\largewhitestar). L’area totale di un parallelepipedo è S. Allora l’area totale del parallelepipedo ottenuto dimezzandone le dimensioni è:

    \[\quad\]

  1. 2S;
  2.  

  3. \dfrac{S}{2};
  4.  

  5. \dfrac{S}{4};
  6.  

  7. S;
  8.  

  9. \dfrac{1}{3}S.

Svolgimento.

Risposta 3.

Se il parallelepipedo ha dimensioni x, y, z, allora la sua area è

    \[ S=2xy+2yz+2xz. \]

Invece, l’area S' di un parallelepipedo ottenuto dimezzandone le dimensioni pari a

    \[ x'\coloneqq\frac{x}{2}, \qquad y'\coloneqq\frac{y}{2} \qquad \makebox{e} \qquad z'\coloneqq\frac{z}{2}, \]

è

    \[ \begin{split} 	S' &=2x'y'+2y'z'+2x'z' \\ 	&=\frac{xy}{2}+\frac{yz}{2}+\frac{xz}{2} \\ 	& =\frac{1}{4}(2xy+2yz+2xz) \\ 	& =\frac{S}{4}. \end{split} \]

La risposta giusta è quindi la 3.


 

Esercizio 7  (\bigstar\largewhitestar\largewhitestar\largewhitestar\largewhitestar). Un’automobile in un concessionario viene venduta ad un certo prezzo durante la sua prima settimana di vendita. Nelle settimane successive, il prezzo dell’automobile viene dimezzato di settimana in settimana finché il suo costo raggiunge una cifra minore o uguale a un quinto del suo valore iniziale. In quel caso, l’automobile viene rivenduta a partire da quella settimana al prezzo iniziale di vendita. In quale settimana conviene comprare l’automobile?

    \[\quad\]

  1. Alla terza settimana;
  2.  

  3. Alla quarta settimana;
  4.  

  5. Dopo 5 settimane;
  6.  

  7. Non conviene mai;
  8.  

  9. Alla seconda settimana.

Svolgimento.

Risposta 1.

Sia P il prezzo iniziale e P_n il prezzo all’n-esima settimana. Allora

    \[ P_n=\frac{P_{n-1}}{2}=\dots=\frac{P}{2^{n-1}}. \]

La settimana per cui conviene comprare equivale al più grande n per cui P_n>\dfrac{P}{5}, ovvero

    \[ \frac{P}{2^{n-1}}>\frac{P}{5} \implies 2^{n-1}<5 \implies n<1+\log_2 5<4. \]

Perciò n=3 è la settimana più conveniente e la risposta esatta è la 1.


 

Esercizio 8  (\bigstar\bigstar\largewhitestar\largewhitestar\largewhitestar). Per quali valori di x è verificata la seguente disequazione?

    \[ \frac{x^2+1}{x^2}\geq 1 \]

    \[\quad\]

  1. Per nessun valore di x;
  2.  

  3. Per tutti i valori di x\in \mathbb R;
  4.  

  5. Per 0<x\leq 1;
  6.  

  7. Per x>0;
  8.  

  9. Per tutti i valori di x tranne x=0.

Svolgimento.

Risposta 5.

Osserviamo che x^2 è sempre positivo e x^2 è sempre minore di x^2+1, perciò il rapporto fra x^2+1 e x^2 supera 1. Dato che stiamo facendo un rapporto, dobbiamo tenere conto che il calcolo ha senso solo quando il denominatore è diverso da 0, perciò solo per x\neq 0. Quindi la disequazione è valida per tutti i valori di x tranne x=0.


 

Esercizio 9  (\bigstar\bigstar\largewhitestar\largewhitestar\largewhitestar). Qual è la probabilità che si verifichi l’evento 2-3-4 in seguito a 3 lanci ordinati di un dado a 4 facce?

    \[\quad\]

  1. \dfrac{1}{4};
  2.  

  3. \dfrac{1}{64};
  4.  

  5. \dfrac{1}{32};
  6.  

  7. \dfrac{1}{3};
  8.  

  9. del cinquanta per cento.

Svolgimento.

Risposta 2.

Gli eventi del lancio di un dado sono fra loro indipendenti, quindi la probabilità è il prodotto delle probabilità di ottenere 2-3-4 in sequenza. Dato che la probabilità in tutti e tre i casi è sempre \dfrac{1}{4}, allora la risposta è \dfrac{1}{4^3}=\dfrac{1}{64}.


 

Esercizio 10  (\bigstar\bigstar\largewhitestar\largewhitestar\largewhitestar). Qual è il massimo numero intero che divide il prodotto di ogni cinquina di numeri interi consecutivi?

    \[\quad\]

  1. 120;
  2.  

  3. 6;
  4.  

  5. 2;
  6.  

  7. 20;
  8.  

  9. 24.

Svolgimento.

Risposta 1.

Consideriamo la prima cinquina:

    \[ 1 \quad 2 \quad 3 \quad 4 \quad 5. \]

Abbiamo quindi che tale numero non può superare 1\cdot 2\cdot 3\cdot 4\cdot 5=120. D’altronde in ogni cinquina di numeri consecutivi vi deve essere un numero multiplo di 5 dato che sono spaziati proprio di 5 in 5. In modo analogo, tra i cinque numeri ci deve essere almeno un multiplo di 3, un multiplo di 4, e infine un multiplo di 2 ma non di 4. Dunque il loro prodotto è divisibile per 5\cdot 4\cdot 3\cdot 2=120.


 

Scarica gli esercizi svolti

Ottieni il documento contenente 120 esercizi risolti, contenuti in 107 pagine ricche di dettagli, per i test di ingresso per ingegneria

 

Esercizio 11  (\bigstar\bigstar\largewhitestar\largewhitestar\largewhitestar). Un cliente deposita in banca 700 euro e ha le seguenti condizioni: 12\% di interesse annuo e 8 euro di spese annue a fine anno. Quanto ha sul conto dopo 3 anni?

    \[\quad\]

  1. 1256,20 euro;
  2.  

  3. 674,23 euro;
  4.  

  5. 956,45 euro;
  6.  

  7. 13455 euro;
  8.  

  9. 728 euro.

Svolgimento.

Risposta 3.

Sia N il deposito iniziale, t gli anni, x\% l’interesse e k le spese annue. Infine sia N_t l’ammontare del conto in banca all’anno t. Quindi N_0=N. Sia c\coloneqq \dfrac{100+x}{100}; allora

    \[ \begin{split} 	N_1 & =N_0+\frac{x}{100}N_0-k =cN-k; \\ 	N_2 & =N_1+\frac{x}{100}N_1-k= cN_1-k=c^2N-(c+1)k; \\	 	N_3 & =N_2+\frac{x}{100}N_2-k= cN_2-k=c^3N-(c^2+c+1)k. \\	 \end{split} \]

Otteniamo quindi per N=700, t=3, x=12, k=8 che il conto del cliente dopo 3 anni ammonta a N_3=956,45 euro.

Nel caso in cui si volesse dedurre una formula per determinare automaticamente N_t in funzione dell’anno t, basta generalizzare il caso di sopra per un qualunque t\geq 1:

    \[ \begin{split} 	N_t & =N_{t-1}+ \frac{x}{100}N_{t-1}-k =cN_{t-1}-k \\ 	& 	= c\left(N_{t-2}+ \frac{x}{100}N_{t-2}-k\right)-k=c^2N_{t-2}-(c+1)k 	\\& =\dots\\&= 	c^tN_0-k\left(\sum_{j=0}^{t-1}c^{j}\right).\\ \end{split} \]

Quindi otteniamo

(1)   \begin{equation*} 	N_t=c^tN_0-k\left(\sum_{j=0}^{t-1}c^{j}\right). \end{equation*}

La formula (1) è stata solamente intuita ma per verificare che sia effettivamente quella corretta invitiamo il lettore a dimostrarla usando il principio di induzione.

Inoltre osserviamo che (1) può essere riscritta in una maniera più compatta usando il fatto che si conosce la somma parziale di una serie geometrica di ragione c:

(2)   \begin{equation*} 	\sum_{j=0}^{t-1}c^{j}=\frac{1-c^t}{1-c}. \end{equation*}

Perciò, sostistuendo la sommatoria in (1) con il valore ottenuto in (2), otteniamo

    \[\begin{split} 	N_t & =c^tN_0-k\frac{1-c^t}{1-c}=\left(N+\frac{k}{1-c}\right)c^t-\frac{k}{1-c}, \end{split} \]

da cui si può di nuovo calcolare che N_3=956,45 euro.


 

Esercizio 12  (\bigstar\bigstar\largewhitestar\largewhitestar\largewhitestar). In un sistema di riferimento cartesiano Oxy, sia r la retta di equazione

    \[ y=\frac{3x+1}{2}. \]

Quale delle seguenti equazioni descrive una retta ortogonale ad r e passante per il punto (1,1)?

    \[\quad\]

  1. y=\dfrac{2}{3}x+\dfrac{5}{3};
  2.  

  3. y=\dfrac{2}{3}x+\dfrac{1}{3};
  4.  

  5. x=1;
  6.  

  7. y=-\dfrac{2}{3}x+\dfrac{5}{3};
  8.  

  9. y=-\dfrac{2-3x}{3}.

Svolgimento.

Risposta 4.

L’equazione di una retta r non parallela all’asse delle ordinate è y=mx+q, dove m è il coefficiente angolare della retta e q è l’intercetta. Il coefficiente angolare di una retta ortogonale ad r è -\dfrac{1}{m}. Quindi una retta ortgonale ad r e passante per un punto P=(x_0,y_0) ha equazione

    \[ y-y_0=-\frac{1}{m}(x-x_0). \]

Nel nostro caso specifico abbiamo m=\dfrac{3}{2} e P=(1,1), perciò la retta cercata ha equazione

    \[ y=-\frac{2}{3}(x-1)+1= -\frac{2}{3}x+\frac{5}{3}. \]

In figura 4 mostriamo le due rette:

    \[\quad\]

    \[\quad\]

Figura 4: la retta r e la sua perpendicolare per il punto (1,1).

    \[\quad\]

    \[\quad\]

La risposta giusta è quindi la 4.


 

Esercizio 13  (\bigstar\largewhitestar\largewhitestar\largewhitestar\largewhitestar). L’equazione 2x+3=12

    \[\quad\]

  1. non ha soluzioni nell’insieme \mathbb Z dei numeri interi;
  2.  

  3. ha sempre almeno una soluzione nell’insieme \mathbb Z;
  4.  

  5. non ha soluzioni nell’inisieme \mathbb Q dei numeri razionali;
  6.  

  7. ha soluzioni nell’insieme \mathbb R dei numeri reali ma non su \mathbb Q;
  8.  

  9. non ha mai soluzione nell’insieme \mathbb R.

Svolgimento.

Risposta 1.

Dall’equazione si ottiene che 2x=9, che su \mathbb Z non può avere soluzioni, dato che 2x è sempre un numero pari e 9 è un numero dispari. Se si ragiona invece su una estensione di \mathbb Z come ad esempio \mathbb Q, una soluzione esiste ed è banalemente x=\dfrac{9}{2}.


 

Esercizio 14  (\bigstar\bigstar\largewhitestar\largewhitestar\largewhitestar). Sia A=\{2,3\} e B=\{\textrm{Juventus}, \textrm{Milan},\textrm{Roma}\}. Quante funzioni non iniettive esistono da A a B?

    \[\quad\]

  1. 2;
  2.  

  3. 4;
  4.  

  5. 6;
  6.  

  7. 9;
  8.  

  9. 3.

Svolgimento.

Risposta 5.

Una funzione definita su due oggetti si può vedere come una collezione di 2 scatole affiancate dove dentro possiamo inserire uno solo dei 3 oggetti del codominio, eventualmente anche ripetuti. Il numero totale di possibili inserimenti è quindi 3\cdot 3=9, che è quindi anche il numero di tutte le possibili funzioni da A a B. Nel caso di funzioni iniettive invece, gli oggetti da inserire non possono essere ripetuti, perciò in totale sono solo 3\cdot 2=6. Il numero di funzioni non iniettive è quindi 9-6=3.

Una soluzione alternativa può essere la seguente. A è composto di due soli elementi, quindi una funzione non iniettiva definita su A è tale per cui i due elementi hanno la stessa immagine. Dato che B consta di 3 elementi, allora ci sono solamente 3 funzioni non iniettive.


 

Esercizio 15  (\bigstar\bigstar\bigstar\largewhitestar\largewhitestar). Quanti sono i possibili valori x nell’intervallo (0, 2\pi) che soddisfanno la seguente equazione?

    \[ \sin(x)\cos(x)=-\frac{1}{3\sqrt{2}}. \]

    \[\quad\]

  1. Non ci sono soluzioni per x\in (0, 2\pi);
  2.  

  3. Ci sono in totale 4 soluzioni distinte;
  4.  

  5. Ci sono in totale 2 soluzioni distinte;
  6.  

  7. C’è una sola soluzione;
  8.  

  9. Ci sono infinite soluzioni nell’intervallo (0, 2\pi).

Svolgimento.

Risposta 2.

Usando la formula di duplicazione del seno, l’equazione data si può riarrangiare come

    \[ 2\sin(x)\cos(x)=-\frac{2}{3\sqrt{2}} \implies \sin(2x)=-\frac{\sqrt{2}}{3}. \]

Dato che la funzione seno è suriettiva nell’intervallo [-1,1] e -\dfrac{\sqrt{2}}{3}\in [-1, 1], allora sicuramente esistono soluzioni dell’equazione. L’intersezione della retta y=-\dfrac{\sqrt{2}}{3} con il grafico della funzione f(x)=\sin(2x) consta di due punti distinti per 0\leq x\leq \pi. Dato che la funzione f è \pi-periodica, allora ci sono esattamente 4 soluzioni distinte per 0\leq x\leq 2\pi (vedi figura 5).

    \[\quad\]

    \[\quad\]

Figura 5: grafico della funzione f(x)=\sin(2x) e intersezione con la retta y=-\dfrac{\sqrt{2}}{3}.

    \[\quad\]

    \[\quad\]

La risposta giusta è quindi la 2.


 

Esercizio 16  (\bigstar\bigstar\largewhitestar\largewhitestar\largewhitestar). Il polinomio x^3-1 è divisibile per:

    \[\quad\]

  1. x^2+x+1;
  2.  

  3. x+1;
  4.  

  5. x;
  6.  

  7. x^2-x+1;
  8.  

  9. è irriducibile.

Svolgimento.

Risposta 1.

Segue dalla scomposizione x^3-1=(x-1)(x^2+x+1). Alternativamente, si può osservare che x^3-1 è divisibile per x-1 per il teorema di Ruffini, in quanto il suo valore per x=1 è nullo. Applichiamo il metodo di Ruffini per dividere x^3 - 1 per x - 1: Otteniamo che x^3 - 1 = (x - 1)(x^2 + x + 1). Dunque il quoziente della divisione è la risposta 1.


 

Esercizio 17  (\bigstar\largewhitestar\largewhitestar\largewhitestar\largewhitestar). Il dominio di esistenza dell’espressione

    \[ f(x)=\log_2\left(\frac{x^2-1}{x-1}\right) \]

è:

    \[\quad\]

  1. (-1, +\infty);
  2.  

  3. (1, +\infty);
  4.  

  5. (-1, 1)\cup (1,+\infty);
  6.  

  7. [-1, 1);
  8.  

  9. l’insieme vuoto.

Svolgimento.

Risposta 3.

La funzione logaritmo in una certa base è definita solamente quando l’argomento è positivo. Inoltre osserviamo che il numeratore dell’argomento si può scomporre come x^2-1=(x-1)(x+1). Supponendo x\neq 1 e dividendo il numeratore per il denominatore x-1 otteniamo che l’argomento è x+1, il quale è positivo solo per x>-1. Ricordando che la funzione del quesito non era ben befinita per x=1, il dominio di esistenza di f è (-1,1)\cup (1,+\infty ). In figura 6 mostriamo il suo grafico:

    \[\quad\]

    \[\quad\]

Figura 6: grafico della funzione f insieme al suo asintoto verticale x=-1.

    \[\quad\]

    \[\quad\]

La risposta corretta è quindi la 3.


 

Esercizio 18  (\bigstar\bigstar\largewhitestar\largewhitestar\largewhitestar). Si hanno a disposizione 7 colori per colorare i vertici di un pentagono, i quali sono numerati da 1 a 5 in senso orario. Quanti sono i modi per colorarli sapendo di non poter colorare più di un vertice dello stesso colore?

    \[\quad\]

  1. 975;
  2.  

  3. 1280;
  4.  

  5. 2520;
  6.  

  7. 412;
  8.  

  9. 476.

Svolgimento.

Risposta 3.

Il primo vertice può essere colorato con tutti e 7 i colori, quindi abbiamo 7 scelte. Per ciascuna di queste scelte, il secondo vertice si può colorare solo con uno dei 6 colori rimanenti, il terzo vertice solo con uno dei 5 colori rimanenti e così via. Moltiplicando tutte queste possiblità si ottiene il numero di modi di colorare i 5 vertici ordinati con 7 colori:

    \[ 	7\cdot 6\cdot 5\cdot 4\cdot 3=2520.  	\]

In figura 7 diamo un esempio di colorazione del pentagono:

    \[\quad\]

    \[\quad\]

Figura 7: esempio di colorazione dei vertici del pentagono usando 5 dei 7 colori disponbili.

    \[\quad\]

    \[\quad\]

La risposta giusta è quindi la 3.


 

Esercizio 19  (\bigstar\bigstar\largewhitestar\largewhitestar\largewhitestar). La retta nel piano Oxy di equazione y=2x e la circonferenza di equazione x^2+y^2-10x+5=0 sono:

    \[\quad\]

  1. non incidenti;
  2.  

  3. incidenti in più di due punti;
  4.  

  5. incidenti in due punti;
  6.  

  7. tangenti;
  8.  

  9. nessuna delle precedenti.

Svolgimento.

Risposta 4.

Sostituendo l’equazione della retta in quella della circonferenza otteniamo

    \[ x^2+4x^2-10x+5=0 \iff 5(x-1)^2=0 \iff x=1. \]

Dato che la loro intersezione consta di un solo punto di molteplicità 2, allora sono fra loro tangenti ed il punto di tangenza è P=(1,2) (vedi figura 8).

    \[\quad\]

    \[\quad\]

Figura 8: retta y=2x tangente alla circonferenza di centro (5,0).

    \[\quad\]

    \[\quad\]

La risposta giusta è quindi la 4.


 

Esercizio 20  (\bigstar\bigstar\largewhitestar\largewhitestar\largewhitestar). Quali delle seguenti funzioni verifica che

    \[ f'(x)=\frac{1}{f(x)} \]

nel dominio di esistenza di f ed f'?
sono:

    \[\quad\]

  1. f(x)=2^x;
  2.  

  3. f(x)=\dfrac{1}{x};
  4.  

  5. f(x)=x^2;
  6.  

  7. f(x)=\sqrt{2}\sqrt{x};
  8.  

  9. nessuna delle precedenti.

Svolgimento.

Risposta 4.

Nel caso 4 abbiamo che

    \[ f'(x)=\sqrt{2}\frac{1}{2}x^{\frac{1}{2}-1}=\frac{1}{\sqrt{2}\sqrt{x}}=\frac{1}{f(x)} \qquad \forall x>0. \]

Scrivendo le espressioni delle derivate dei restanti casi, si osserva che esse non sono uguali a \dfrac{1}{f(x)}.


 

Esercizio 21  (\bigstar\bigstar\largewhitestar\largewhitestar\largewhitestar). Un partecipante ad un test di ingresso ha risposto correttamente ad 11 quesiti avendo una media di 24 punti. Al 12-esimo quesito lo studente sbaglia domanda e prende -2 punti. Quanto è la nuova media del punteggio?

    \[\quad\]

  1. Circa 21,8 punti;
  2.  

  3. 20 punti;
  4.  

  5. 18 punti;
  6.  

  7. Circa 23,4 punti;
  8.  

  9. 22 punti.

Svolgimento.

Risposta 1.

La media del punteggio finale è

    \[ \frac{p_1+\cdots + p_{12}}{12}=\frac{p_1+\cdots+p_{11}}{12}+\frac{p_{12}}{12}=\frac{11}{12}\frac{p_1+\cdots+p_{11}}{11}+\frac{p_{12}}{12}=\frac{11}{12}\cdot 24-\frac{1}{6}=\frac{131}{6}\approx 21,8. \]

Perciò la risposta esatta è la 1.


 

Esercizio 22  (\bigstar\bigstar\largewhitestar\largewhitestar\largewhitestar). Ordina in ordine crescente i seguenti numeri: x=0,7, y=\dfrac{12}{17}, z=\ln 4, t=\sqrt{e}.

    \[\quad\]

  1. x,t,y,z;
  2.  

  3. x,t, z,y;
  4.  

  5. y,t,x,z;
  6.  

  7. x,z,y,t;
  8.  

  9. x,y,z,t.

Svolgimento.

Risposta 5.

x=0,7=\dfrac{7}{10}>\dfrac{12}{17} se e solo se 119=17\cdot 7>12\cdot 10=120 che è falso, quindi x<y. \ln 4 è poco più di 1 in quanto e è circa 2,71, quindi x<y<z. La 1 viene esclusa perché t<y<1, mentre t>1 perché è la radice di un numero maggiore di 1. Quindi la risposta corretta è la 5.


 

Esercizio 23  (\bigstar\bigstar\largewhitestar\largewhitestar\largewhitestar). L’espressione \left(\sin(\dfrac{\pi}{12})+\cos(\dfrac{\pi}{12})\right)^2 è uguale a:

    \[\quad\]

  1. \dfrac{3}{5};
  2.  

  3. \dfrac{11}{12};
  4.  

  5. 2;
  6.  

  7. \dfrac{3}{2};
  8.  

  9. \dfrac{9}{12}.

Svolgimento.

Risposta 4.

Sviluppiamo il quadrato:

    \[ \left(\sin\left(\frac{\pi}{12}\right)+\cos\left(\frac{\pi}{12}\right)\right)^2=\sin^2\left(\frac{\pi}{12}\right)+\cos^2\left(\frac{\pi}{12}\right)+2\sin\left(\frac{\pi}{12}\right)\cos\left(\frac{\pi}{12}\right)=1+\sin \left(\frac{\pi}{6}\right)=1+\frac{1}{2}=\frac{3}{2}. \]

Nella seconda disuguaglianza si è usata l’identità fondamentale della trigonometria e la formula di duplicazione del seno:

    \[ \sin^2\left(\frac{\pi}{12}\right)+\cos^2\left(\frac{\pi}{12}\right)=1 \qquad \makebox{e} \qquad \sin\left(\frac{\pi}{6}\right)=2\sin\left(\frac{\pi}{12}\right)\cos\left(\frac{\pi}{12}\right). \]

La risposta corretta è quindi la 4.


 

Esercizio 24  (\bigstar\bigstar\bigstar\largewhitestar\largewhitestar). L’età media tra i partecipanti ad un concorso è di 19 anni. Se l’età media degli uomini è di 24 anni e quella delle donne 18, qual è il rapporto fra il numero di donne ed il numero di uomini?

    \[\quad\]

  1. Le donne sono i \dfrac{5}{4} degli uomini;
  2.  

  3. Le donne sono 5 volte gli uomini;
  4.  

  5. Le donne sono la metà degli uomini;
  6.  

  7. Le donne sono 3 volte gli uomini;
  8.  

  9. Le donne sono 7 volte gli uomini.

Svolgimento.

Risposta 2.

Sia N_m il numero di uomini ed N_f il numero di donne. Sia S_m e S_f le somme delle età degli uomini e delle donne rispettivemente. Allora si ha

    \[ 19=\frac{S_m+S_f}{N_m+N_f}, \qquad 24=\frac{S_m}{N_m}, \qquad \makebox{e} \qquad 18=\frac{S_f}{N_f}. \]

Quindi

    \[ 19(N_m+N_f)=24N_m+18N_f, \]

da cui N_f=5N_m.

La risposta corretta è quindi la 2.


 

Esercizio 25  (\bigstar\largewhitestar\largewhitestar\largewhitestar\largewhitestar). La diagonale di una faccia di un cubo è d. Se si ingrandisce il cubo triplicando d, allora il suo volume:

    \[\quad\]

  1. è 27 volte più grande;
  2.  

  3. è 9 volte più grande;
  4.  

  5. è 3 volte più grande;
  6.  

  7. è 12 volte più grande;
  8.  

  9. è 6 volte più grande.

Svolgimento.

Risposta 1.

Usando il teorema di Pitagora, si osserva che il rapporto tra la diagonale della faccia di un cubo e la lunghezza l di un lato è \sqrt{2} (vedi figura 9). In particolare, se la diagonale triplica, allora triplica anche il lato. Quindi il volume del nuovo cubo sarà

    \[ (3l)^3=27l^3, \]

che equivale a 27 volte il volume del cubo originario.

    \[\quad\]

    \[\quad\]

Figura 9: cubo di lato l e quello di dimensioni triplicate.

    \[\quad\]

    \[\quad\]

La risposta corretta è quindi la 1.


 

Esercizio 26  (\bigstar\bigstar\largewhitestar\largewhitestar\largewhitestar). Sono dati tre numeri non nulli x,y,z e si sa che x è la terza parte di z mentre z è 6 volte y. Quanto vale il quoziente fra il doppio di x e la metà di y?

    \[\quad\]

  1. 8;
  2.  

  3. 9;
  4.  

  5. \dfrac{4}{3};
  6.  

  7. 54;
  8.  

  9. 12.

Svolgimento.

Risposta 1.

Dal testo del quesito deduciamo che z=3x e z=6y, ovvero x=2y. Allora

    \[ \frac{2x}{\dfrac{y}{2}}=4\cdot 2\frac{y}{y}=8. \]

La risposta corretta è quindi la 1.


 

Esercizio 27  (\bigstar\bigstar\bigstar\largewhitestar\largewhitestar). Nel piano cartesiano Oxy l’insieme dei punti (x,y) che verificano l’equazione x^2+25y^2-10xy=0 è una:

    \[\quad\]

  1. circonferenza;
  2.  

  3. retta;
  4.  

  5. parabola;
  6.  

  7. ellisse;
  8.  

  9. nessuna delle precedenti.

Svolgimento.

Risposta 2.

Osserviamo che l’equazione si può scrivere come un quadrato perfetto:

    \[ x^2+25y^2-10xy=(x-5y)^2=0. \]

Quindi l’insieme dei punti (x, y) che verificano tale equazione è l’insieme dei punti che verificano

    \[ x-5y=0, \]

ovvero una retta nel piano (vedi figura 10).

    \[\quad\]

    \[\quad\]

Figura 10: grafico della retta x=5y.

    \[\quad\]

    \[\quad\]

La risposta corretta è quindi la 2.


 

Esercizio 28  (\bigstar\largewhitestar\largewhitestar\largewhitestar\largewhitestar). Un hotel chiede 40 euro a notte per le prime 15 notti, se si sceglie un pacchetto di pernottamenti di più di 15 notti. Inoltre, per ogni notte successiva alla 15-esima, si applica uno sconto ulteriore del 23\%. Quanto costa in totale il pernottamento se si prenotano 52 notti?

    \[\quad\]

  1. 980 euro;
  2.  

  3. 1326,7 euro;
  4.  

  5. 1739,6 euro;
  6.  

  7. 1121 euro;
  8.  

  9. 598 euro.

Svolgimento.

Risposta 3.

Ciascuna delle prime 15 notti costa 40 euro, per un totale di 15\cdot 40=600 euro. Invece ciascuna delle 52-15=37 notti restanti costerà 40-40\dfrac{23}{100}=\dfrac{2\cdot 77}{5} euro. Quindi il totale speso è di 600+ 37\cdot \dfrac{2\cdot 77}{5}=1739,6 euro.


 

Esercizio 29  (\bigstar\bigstar\bigstar\largewhitestar\largewhitestar). La somma dei primi 157 numeri naturali consecutivi maggiori di 100 è pari a:

    \[\quad\]

  1. 32476;
  2.  

  3. 12453;
  4.  

  5. 21567;
  6.  

  7. 26451;
  8.  

  9. 28103.

Svolgimento.

Risposta 5.

Carl Friedrich Gauss trovò una formula per calcolare la somma S(n) dei primi n numeri consecutivi:

    \[ S(n)=\frac{n(n+1)}{2}.  \]

Calcolare la somma dei numeri da 101 a 257 equivale a calcolare la differenza fra la somma dei primi 257 numeri e la somma dei primi 100 numeri:

    \[ S(257)- S(100)=\frac{257\cdot 258}{2}-\frac{101\cdot 100}{2}=129\cdot 257-101\cdot 50=28103. \]

Perciò la risposta corretta è la 5.


 

Esercizio 30  (\bigstar\largewhitestar\largewhitestar\largewhitestar\largewhitestar). Nell’insieme dei numeri reali, la disequazione x^4<-16 è verificata per:

    \[\quad\]

  1. infiniti valori;
  2.  

  3. -2\leq x\leq 2;
  4.  

  5. x<-2 oppure x>2;
  6.  

  7. nessun valore di x;
  8.  

  9. -2< x< 2.

Svolgimento.

Risposta 4.

x^4 non è mai un numero negativo, per cui non può essere minore di -16.


 

Esercizio 31  (\bigstar\bigstar\largewhitestar\largewhitestar\largewhitestar). Il massimo comune divisore ed il minimo comune multiplo tra i numeri 12, 40, 60, 120 sono rispettivamente:

    \[\quad\]

  1. 2, 50;
  2.  

  3. 4, 50;
  4.  

  5. 6, 120;
  6.  

  7. 4, 120;
  8.  

  9. 6, 60.

Svolgimento.

Risposta 4.

Tali numeri si scompogono in fattori rispettivamente come segue:

    \[ 2^2\cdot 3 \qquad 2^3\cdot 5 \qquad 2^2\cdot 3 \cdot 5 \qquad 2^3\cdot 3 \cdot 5. \]

Quindi il massimo comune divisore è 2^2=4 e il minimo comune multiplo è 2^3\cdot 3 \cdot 5=120.


 

Esercizio 32  (\bigstar\bigstar\bigstar\largewhitestar\largewhitestar). Dato k>0, allora le equazioni x^2+y^2-2kx=0 nel piano cartesiano Oxy descrivono al variare di k:

    \[\quad\]

  1. un fascio di circonferenze concentriche;
  2.  

  3. un fascio di circonferenze tangenti alla retta x=0;
  4.  

  5. un fascio di circonferenze tangenti alla retta y=0;
  6.  

  7. un fascio di circonferenze generato da due circonferenze date;
  8.  

  9. nessuna delle precedenti.

Svolgimento.

Risposta 2.

Per ogni k fissato, il luogo dei punti (x,y) che verificano l’equazione x^2+y^2-2kx=0 è una circonferenza centrata nel punto (k,0) e di raggio \vert k \vert\geq 0.

Osserviamo inoltre che la retta x=0 interseca ogni circonferenza del fascio in un solo punto P:

    \[ 0^2+y^2-2k\cdot 0=0 \iff y=0 \iff P=(0,0). \]

    \[\quad\]

    \[\quad\]

Figura 11: fascio di circonferenze tangenti alla retta x=0.

    \[\quad\]

    \[\quad\]

Perciò l’equazione è quella di un fascio di circonferenze tangenti alla retta di equazione x=0 (vedi figura 11). Pertanto la riposta coretta è la 2.


 

Esercizio 33  (\bigstar\bigstar\largewhitestar\largewhitestar\largewhitestar). Dire quale tra le seguenti terne di numeri non può descrivere le lunghezze dei lati di un triangolo:

    \[\quad\]

  1. 6, 5, 7;
  2.  

  3. 7, 13 ,18;
  4.  

  5. 5, 7, 1;
  6.  

  7. 5, 12, 14;
  8.  

  9. 5, 13, 14.

Svolgimento.

Risposta 3.

In un qualunque triangolo la somma di due lati deve essere sempre maggiore del lato rimanente (vedi figura 12). In formule, dato un triangolo di lati a,b,c, allora c<a+b.

Una spiegazione può essere la seguente. Il lato di lunghezza c è una porzione di linea retta, mentre i lati di lunghezza a e b descrivono una traiettoria (o meglio, una spezzata) che non è in linea retta. Pertanto la lunghezza della spezzata deve essere maggiore della lunghezza della porzione di linea retta.

Nel nostro caso specifico, possiamo notare che 1+5=6<7, quindi la terna 3 non è ammissibile.

    \[\quad\]

    \[\quad\]

Figura 12: triangolo di lati a,b e c.

    \[\quad\]

    \[\quad\]

La risposta esatta è quindi la 3.


 

Esercizio 34  (\bigstar\bigstar\largewhitestar\largewhitestar\largewhitestar). Un triangolo isoscele ha perimetro lungo 13 cm. Allora i suoi lati possono essere lunghi:

    \[\quad\]

  1. 2, 2, 9;
  2.  

  3. 3, 3, 7;
  4.  

  5. 4, 4, 5;
  6.  

  7. 4, 4, 6;
  8.  

  9. 5, 5, 5.

Svolgimento.

Risposta 3.

1 e 2 non sono una terna di numeri che possono essere i lati di un triangolo, dato che 2+2=4<9 e 3+3=6<7. Infatti la lunghezza di ogni lato di un triangolo è sempre minore della somma dei lati rimanenti (si veda l’esercizio 33 per una spiegazione). Inoltre anche 4 ed 5 sono da escludere in quanto il loro perimetro non è 13 cm. La riposta esatta è quindi la 3.


 

Esercizio 35  (\bigstar\largewhitestar\largewhitestar\largewhitestar\largewhitestar). Quale tra i seguenti numeri non è primo?

    \[\quad\]

  1. 2713;
  2.  

  3. 4591;
  4.  

  5. 743;
  6.  

  7. 937;
  8.  

  9. 639.

Svolgimento.

Risposta 5.

Ricordiamo che un numero è divisibile per 3 solo se la somma delle sue cifre è divisibile per 3. Ci si può convincere di questo scrivendo un numero n=x_kx_{k-1}\cdots x_0, dove x_i indicano le sue cifre decimali:

    \[ n=x_k 10^k+\cdots + x_{1}10+x_0. \]

Il numero 10 si può scrivere come 3\cdot 3+1, e in generale il numero 10^k si può scrivere come (3\cdot 3+1)^k=3t_k+1 per un qualche numero intero t_k dipendente da k. Quindi il numero n può essere scritto come

    \[ n=x_k 10^k+\cdots + x_{1}10+x_0=3(t_kx_k+\dots +3x_1)+(x_k+\dots +x_1). \]

Da qui si deduce che n è divisibile per 3 solo se anche la somma delle sue cifre x_k+\dots +x_1 è divisibile per 3.

Nel caso del quesito, osserviamo che la somma delle cifre di 639 è 18, che è divisibile per 3. Quindi 639 è divisibile per 3 e non è un numero primo. La risposta corretta è quindi la 5.


 

Esercizio 36  (\bigstar\bigstar\largewhitestar\largewhitestar\largewhitestar). Il sistema di equazioni

    \[ \begin{cases} 	x+3y=1 \\ 	7x+21y=7 \end{cases} \]

    \[\quad\]

  1. ammette infinite soluzioni;
  2.  

  3. ammette una sola soluzione;
  4.  

  5. non ammette alcuna soluzione;
  6.  

  7. è risolvibile solo per x>0;
  8.  

  9. è risolvibile solo per x=0.

Svolgimento.

Risposta 1.

Osserviamo che la seconda equazione è 7 volte la prima. In altre parole, è ridondante e non serve per determinare lo spazio delle soluzioni del sistema, che è semplicemente l’insieme delle coppie (x,y) per cui x=-3y+1, ovvero l’insieme delle coppie (-3y+1,y) al variare di y nell’insieme dei numeri reali.


 

Esercizio 37  (\bigstar\bigstar\bigstar\largewhitestar\largewhitestar). La probabilità che lanciando 4 dadi a 4 facce si ottengano 4 risultati identici (tutti 1 o tutti 2 o tutti 3 o tutti 4) è:

    \[\quad\]

  1. \dfrac{1}{256};
  2.  

  3. \dfrac{1}{16};
  4.  

  5. 0;
  6.  

  7. \dfrac{1}{64};
  8.  

  9. 1.

Svolgimento.

Risposta 4.

La probabilità di ottenere uno stesso risultato x\in\{1,2,3,4\} è sempre la stessa ed uguale a \dfrac{1}{4^4}, dato che gli eventi del lancio dei dadi sono fra loro indipendenti. Dato che i tre possibili eventi sono disgiunti, allora la probabilità cercata è \dfrac{1}{4^4}+\dfrac{1}{4^4}+\dfrac{1}{4^4}+\dfrac{1}{4^4}=\dfrac{1}{4^3}=\dfrac{1}{64}, per il teorema della probabilità totale.


 

Esercizio 38  (\bigstar\bigstar\largewhitestar\largewhitestar\largewhitestar). Nel campo dei numeri reali \mathbb R, la disequazione \vert x^2+1\vert <1

    \[\quad\]

  1. è verificata per ogni valore x;
  2.  

  3. non è verificata per alcun valore di x;
  4.  

  5. è verificata per ogni x>0;
  6.  

  7. è verificata per ogni x<0;
  8.  

  9. ha una sola soluzione.

Svolgimento.

Risposta 2.

Osserviamo che x^2+1 è sempre positivo, quindi il suo valore assoluto è sempre x^2+1 e dobbiamo risolvere

    \[ x^2+1<1\implies x^2<0 \]

che è impossibile.


 

Esercizio 39  (\bigstar\bigstar\bigstar\largewhitestar\largewhitestar). Siano P e p i perimetri di due triangoli equilateri, rispettivamente circoscritto e inscritto ad una circonferenza di raggio r>0. Allora:

    \[\quad\]

  1. non c’è alcuna relazione fra P e p;
  2.  

  3. P=\sqrt{3}p;
  4.  

  5. p è la terza parte di P;
  6.  

  7. P è 2 volte p;
  8.  

  9. P è 4 volte p.

Svolgimento.

Risposta 4.

Senza perdita di generalità, si possono posizionare i due triangoli in modo che due dei loro lati siano paralleli (vedi figura 13).

    \[\quad\]

    \[\quad\]

Figura 13: triangoli equilateri, uno circoscritto e l’altro inscritto alla circonferenza.

    \[\quad\]

    \[\quad\]

Vediamo due metodi che si possono adottare per risolvere il problema, quello di similitudine dei triangoli e quello che fa uso dei teoremi di trigonometria sui triangoli rettangoli insieme al teorema della corda.

Analizziamo il primo caso. Il triangolo \triangle{AOH'} è equilatero in quanto OA e OH' hanno lunghezza r e \angle{AOH'}=\dfrac{\pi}{3}. Allora AH è anche la mediana di \angle{AOH'} e pertanto OH=HH', da cui OH=\dfrac{r}{2}.

Adesso osserviamo che i due trinagoli rettangoli \triangle{AOH} e \triangle{A'OH'} sono simili, pertanto vale la proporzione:

    \[ \frac{A'H'}{OH'}=\frac{AH}{OH} \implies A'H'=2AH. \]

Dato che A'H' e AH sono rispettivemente la metà di A'B' e AB, allora A'B'=2AB. Di conseguenza, il perimetro di \triangle{A'B'C'} è il doppio di \triangle{ABC}.

Vediamo adesso il secondo metodo. Dal triangolo rettangolo \triangle{A'OH'} si deduce che

    \[ A'H'=OH'\tan\left(\frac{\pi}{3}\right) \implies A'B'=2A'H'=2r\sqrt{3}. \]

Invece, applicando il toerema della corda sul lato AB si ottiene che

    \[ AB=2r\sin\left(\frac{\pi}{3}\right)= r\sqrt{3}. \]

Quindi A'B'=2AB e pertanto un perimetro è il doppio dell’altro.


 

Esercizio 40  (\bigstar\bigstar\largewhitestar\largewhitestar\largewhitestar). Supponendo che a,b>0, che a^2+b^2=11 e ab=7, allora quanto vale a+b?

    \[\quad\]

  1. \dfrac{3}{2};
  2.  

  3. 8;
  4.  

  5. 5;
  6.  

  7. dipende dalla scelta di a e da b;
  8.  

  9. non ci sono abbastanza dati per calcolare a+b.

Svolgimento.

Risposta 3.

Consideriamo il quadrato

    \[ (a+b)^2=a^2+b^2+2ab=11+14=25 \implies a+b=5, \]

dato che per ipotesi a,b>0 e quindi a+b>0.

    \[\quad\]

    \[\quad\]

Figura 14: visualizzazione geometrica di come può essere suddiviso l’area di un quadrato di lato a+b.

    \[\quad\]

    \[\quad\]

Quindi la risposta corretta è la 3.


 

Esercizio 41  (\bigstar\bigstar\bigstar\largewhitestar\largewhitestar). In una festa di compleanno il 17 per cento degli invitati mangia panini, il 23 per cento mangia noccioline, il 37 per cento mangia pizzette. Inoltre, il 6 per cento mangia sia panini che noccioline, il 19 per cento mangia sia noccioline che pizzette e il 12 per cento mangia sia panini che pizzette. Infine il 2 per cento degli invitati mangia tutti gli stuzzichini. Qual è la percentuale di invitati che non mangia nulla?

    \[\quad\]

  1. 58\%;
  2.  

  3. 47\%;
  4.  

  5. 12\%;
  6.  

  7. 19\%;
  8.  

  9. 1\%.

Svolgimento.

Risposta 1.

Dato che 2\% è il numero di chi mangia tutti gli stuzzichini, allora:

    \[\quad\]

  • il 6\%-2\%=4\% mangia solo sia panini che noccioline;
  •  

  • il 19\%-2\%=17\% mangia solo sia noccioline che pizzette;
  •  

  • il 12\%-2\%=10\% mangia solo sia panini che pizzette.

Invece:

    \[\quad\]

  • il 17\%-4\%-10\%-2\%=1\% mangia solo panini;
  •  

  • il 23\%-4\%-17\%-2\%=0\% mangia solo noccioline;
  •  

  • il 37\%-17\%-10\%-2\%=8% mangia solo pizzette.

Se ne deduce che chi mangia qualcosa è il 1%+0%+8%+4%+17%+10%+2%=42%, quindi chi non mangia nulla è il 58\%.


 

Esercizio 42  (\bigstar\bigstar\largewhitestar\largewhitestar\largewhitestar). Qual è il polinomio tra i seguenti che ha più radici reali distinte?

    \[\quad\]

  1. x^2+2x+2;
  2.  

  3. x^2-2x+1;
  4.  

  5. x^2-5x+6;
  6.  

  7. x^3+3x^2+3x+1;
  8.  

  9. (x-1)(x-2)(x-3).

Svolgimento.

Risposta 5.

Il polinomio 5 ha tre radici distinte mentre i polinomi 1, 2, 3 sono di secondo grado e non possono quindi avere più di due radici reali distinte. Infine il polinomio 4 è (x+1)^3, che ha una sola radice distinta x=-1. Perciò la risposta corretta è la 5.


 

Esercizio 43  (\bigstar\largewhitestar\largewhitestar\largewhitestar\largewhitestar). Un cerchio ed un quadrato hanno la stessa la area. Si può quindi dedurre che:

    \[\quad\]

  1. i perimetri sono uguali;
  2.  

  3. il perimetro del quadrato è minore di quello del cerchio;
  4.  

  5. il rapporto tra il perimetro del cerchio e quello della quadrato è 2\sqrt{\pi};
  6.  

  7. il rapporto tra il perimetro del quadrato e quello del cerchio è \dfrac{2}{\sqrt{\pi}};
  8.  

  9. nessuna delle precedenti.

Svolgimento.

Risposta 4.

L’area del quadrato è A_q=l^2 mentre l’area di un cerchio è A_c=r^2\pi, perciò

    \[ 	A_q=A_c \implies l^2=r^2\pi\implies l=r\sqrt{\pi},\]

quindi

    \[ 	P_q=4l=4r\sqrt{\pi}=2\frac{P_c}{\sqrt{\pi}} \implies \frac{P_q}{P_c}=\frac{2}{\sqrt{\pi}}. 	\]

    \[\quad\]

    \[\quad\]

Figura 15: quadrato e cerchio aventi area uguale.

    \[\quad\]

    \[\quad\]

Quindi la risposta corretta è la 4.


 

Esercizio 44  (\bigstar\bigstar\largewhitestar\largewhitestar\largewhitestar). L’uguaglianza \sqrt{(b+1)^2}=b+1 è verificata per

    \[\quad\]

  1. b<-1;
  2.  

  3. b\leq -1;
  4.  

  5. b\geq -1;
  6.  

  7. b>-1;
  8.  

  9. b=-1.

Svolgimento.

Risposta 3.

La radice di un quadrato è il valore assoluto dell’argomento, dunque l’equazione è equivalente a \vert b+1\vert =b+1. Per definizione di valore assoluto, \vert b+1\vert è b+1 se e solo se b+1\geq 0, che è equivalente a b\geq -1.

La risposta esatta è quindi la 3.


 

Esercizio 45  (\bigstar\bigstar\bigstar\largewhitestar\largewhitestar). Sia f\colon \mathbb R\to \mathbb R una funzione reale di variabile reale. Allora la funzione g\colon \mathbb R\to \mathbb R definita da g(x)\coloneqq f(x^2)

    \[\quad\]

  1. è una funzione dispari;
  2.  

  3. è una funzione pari;
  4.  

  5. non è una funzione;
  6.  

  7. si annulla negli stessi punti in cui si annulla f;
  8.  

  9. non si annulla mai.

Svolgimento.

Risposta 2.

La funzione g è sempre pari in quanto

    \[ 	g(-x)=f((-x)^2)=f(x^2)=g(x) \qquad \forall x\in \mathbb R,  	\]

In figura 16, mostriamo un esempio con f(x)=\sin(x).

    \[\quad\]

    \[\quad\]

Figura 16: grafici delle funzioni f e g.

    \[\quad\]

    \[\quad\]

La risposta giusta è quindi la 2.


 

Esercizio 46  (\bigstar\bigstar\bigstar\largewhitestar\largewhitestar). La funzione f\colon \mathbb R\to \mathbb R^3 definita da

    \[ 		f(t)=\left(\frac{\sqrt{2}}{2}, \frac{\sqrt{2}}{2}\sin(t), \frac{\sqrt{2}}{2}\cos(t)\right) \qquad \forall t\in\mathbb R 		\]

    \[\quad\]

  1. descrive una traiettoria sulla sfera di \mathbb R^3 centrata nell’orgine e raggio 1;
  2.  

  3. perde di significato per alcuni valori di t\in\mathbb R;
  4.  

  5. è una traiettoria contenuta nel piano y=z di \mathbb R^3;
  6.  

  7. è una traiettoria contenuta nel piano x=z di \mathbb R^3;
  8.  

  9. nessuna delle precedenti.

Svolgimento.

Risposta 1.

Osserviamo che la norma di ogni punto f(t) è 1:

    \[ 	\|f(t)\|^2=\frac{1}{2}(1+\sin^2(t)+\cos^2(t)) =1, 	\]

quindi la traiettoria giace sulla sfera unitaria di \mathbb R^3 (vedi figura 17).

    \[\quad\]

    \[\quad\]

Figura 17: sfera centrata nell’origine e di raggio 1 insieme alla traiettoria descritta da f.

    \[\quad\]

    \[\quad\]

Perciò la risposta esatta è la 1.


 

Esercizio 47  (\bigstar\bigstar\largewhitestar\largewhitestar\largewhitestar). Quale realzione sussiste fra \log_b(a) e \log_c(a)?

    \[\quad\]

  1. \log_b(a)=\dfrac{\log_c(a)}{\log_b(c)};
  2.  

  3. \log_c(a)=\dfrac{\log_b(a)}{\log_b(c)};
  4.  

  5. non esiste alcuna relazione fra loro;
  6.  

  7. \log_b(a)\log_b(c)\log_c(a)=1;
  8.  

  9. sono uguali.

Svolgimento.

Risposta 2.

La formula è quella del cambio di base del logaritmo, ma volendo la si può facilemente ricavare:

    \[ 	b^{\log_b(a)}=a=c^{\log_c(a)}=\left(b^{\log_b(c)}\right)^{\log_c(a)}=b^{\log_b(c)\log_c(a)}.  	\]

Uguagliando gli esponenti del primo e dell’ultimo membro otteniamo che la 2 è la risposta corretta.


 

Esercizio 48  (\bigstar\bigstar\bigstar\largewhitestar\largewhitestar). Una parabola

    \[\quad\]

  1. è il luogo dei punti del piano che distano equamente da un certo punto e una certa retta data;
  2.  

  3. è il luogo dei punti del piano che distano equamente da una coppia di punti distinti;
  4.  

  5. è il luogo dei punti del piano la cui somma delle distanze da una coppia di punti distinti è costante;
  6.  

  7. è il luogo dei punti del piano la cui differenza delle distanze da una coppia di punti distinti è costante;
  8.  

  9. nessuna delle precedenti.

Svolgimento.

Risposta 1.

Si tratta proprio della definizione di parabola in geometria analitica. Partendo da un punto ed una retta (per semplicità parallela all’asse delle ascisse o all’asse delle ordinate) si ricava da tale definizione l’equazione cartesiana della parabola, che equivale a

    \[ 	y=ax^2+by^2+c \qquad \makebox{oppure} \qquad x=ay^2+by+c, \quad a\neq0.  	\]

    \[\quad\]

    \[\quad\]

Figura 18: costruzione di una parabola a partire dal suo fuoco (0,1) e dalla sua direttrice y=-1.

    \[\quad\]

    \[\quad\]

La risposta corretta è quindi la 1.


 

Esercizio 49  (\bigstar\bigstar\largewhitestar\largewhitestar\largewhitestar). Data un’espressione y(x)=bx^2+ax+1, allora se x triplica, di quanto aumenta y?

    \[\quad\]

  1. 8bx^2+2ax;
  2.  

  3. 9bx^2+3ax+1;
  4.  

  5. 8bx+1;
  6.  

  7. 8bx^2;
  8.  

  9. 2ax.

Svolgimento.

Risposta 1.

y(x) valutata in 3x equivale a

    \[ 	y(3x)=b(3x)^2+a(3x)+1=9bx^2+3ax+1=(bx^2+ax+1)+ 8bx^2+2ax=y(x)+8bx^2+2ax 	\]

perciò y aumenta di 8bx^2+2ax.


 

Esercizio 50  (\bigstar\bigstar\bigstar\bigstar\largewhitestar). Il numero 3261531 in base 7 è divisibile per:

    \[\quad\]

  1. 3;
  2.  

  3. 2;
  4.  

  5. 5;
  6.  

  7. 6;
  8.  

  9. nessuna delle precedenti.

Svolgimento.

Risposta 1.

Dato che 7 è congruo a 1 modulo 3, cioè 7 diviso 3 ha resto di 1, allora ogni numero in base 7 è divisibile per 3 semplicemente se la somma delle sue cifre è divisibile per 3. Infatti, se un numero n in base 7 è rappresentato dalle cifre x_kx_{k-1}\cdots x_0, ovvero

    \[ 	n= x_k7^k+x_{k-1}7^{k-1}+\cdots + x_0, 	\]

allora visto che 7^j=3t_j+1 per un certo numero intero t_j dipendente da j, si ha

    \[ 	n=x_k7^k+x_{k-1}7^{k-1}+\cdots + x_0=3(t_kx_k+t_{k-1}x_{k-1}+\cdots 2x_1)+(x_k+x_{k-1}+\cdots x_0). 	\]

Da qui si deduce facilemente che n è divisibile per 3 solo se la somma x_k+x_{k-1}+\dots + x_0 è divisibile per 3.

Nel nostro caso specifico abbiamo

    \[ 	3+2+6+1+5+3+1=21, 	\]

che è divisibile per 3. Con ragionamenti simili si vede che il numero non è divisibile per 2 (e quindi neanche per 6) e per 5.


 

Esercizio 51  (\bigstar\bigstar\largewhitestar\largewhitestar\largewhitestar). Dato a>0, a\neq 1, il logaritmo \log_a(x) di un numero x in base a è:

    \[\quad\]

  1. quel numero che si ottiene elevando a per x;
  2.  

  3. quel numero che si ottiene elevando x per a
  4.  

  5. quel numero che elevato ad a è uguale ad x;
  6.  

  7. l’esponente che devi dare ad a per ottenere x;
  8.  

  9. nessuna delle precedenti.

Svolgimento.

Risposta 4.

La 4 è proprio la definizione di logaritmo.


 

Esercizio 52  (\bigstar\largewhitestar\largewhitestar\largewhitestar\largewhitestar). Risolvere la disequazione

    \[ 			\frac{x^2-1}{x^3-x^2+x}\geq 0. 			\]

    \[\quad\]

  1. -1\leq x<0 oppure x\geq 1;
  2.  

  3. la disequazione vale per ogni valore reale x tranne x=0;
  4.  

  5. x\leq -1 oppure x>1;
  6.  

  7. -1\leq x\leq 1;
  8.  

  9. x=1.

Svolgimento.

Risposta 1.

Studiamo prima i segni del numeratore e del denominatore.

    \[ 	x^2-1\geq 0 \iff x\leq -1 \quad \vee  	\quad x\geq 1. 	\]

Invece, l’espressione x^3-x^2+x si decompone come x\left(x^2-x+1\right). Il discriminante del fattore di secondo grado x^2-x+1 è \Delta=(-1)^2-4\cdot 1\cdot 1=-3<0, perciò il polinomio non ammette radici reali ed il suo segno è sempre positivo. Il segno di x(x^3-x^2+1) è quindi lo stesso di x.

Facendo il prodotto dei segni con l’usuale grafico otteniamo il segno della frazione del quesito:

    \[\quad\]

    \[\quad\]

Pertanto la frazione assume valori non negativi per -1\leq x<0 oppure per x\geq 1.


 

Esercizio 53  (\bigstar\bigstar\largewhitestar\largewhitestar\largewhitestar). Siano a, b, c tre valori reali. Quali delle seguenti affermazioni è vera?

    \[\quad\]

  1. ab+bc\leq a^2c^2;
  2.  

  3. abc\leq ab+bc+ca;
  4.  

  5. ab+bc+ac\leq \dfrac{1}{3}\left(a^2+b^2+c^2\right);
  6.  

  7. ab+bc+ac\leq a^2+b^2+c^2;
  8.  

  9. nessuna delle precedenti.

Svolgimento.

Risposta 4.

Si osserva che

    \[ 	\begin{split} 		\left(a-b\right)^2+\left(a-c\right)^2+\left(b-c\right)^2\geq 0 \implies 2a^2+2b^2+2c^2-2ab-2ac-2bc\geq 0 	\end{split} 	\]

da cui si ottiene immediatamente la 4.

Un modo per dedurre la disugualianza 4 geometricamente è risolvere il seguente problema:

Trovare tra i parallelipedi aventi una diagonale di lunghezza fissa d>0 quello di superficie totale massima.

Dato un parallelipedo di lati a,b,c >0, allora l’area totale A(a,b,c) è 2ab+2bc+2ac. La lunghezza della diagonale in funzione dei lati si può calcoalre applicando due volte il Teorema di Pitagora, da cui si ottiene:

(3)   \begin{equation*} 		a^2+b^2+c^2=d^2.  	\end{equation*}

Pertanto, risolvere il problema equivale a trovare il massimo della funzione in tre variabili A(a,b,c) sul vincolo a^2+b^2+c^2=d^2. Esistono tecniche avanzate per determinare il massimo assoluto di una funzione lungo un certo vincolo (es. metodo dei Moltiplicatori di Lagrange), ma non andremo in dettaglio. Comunque, intuitivamente si può dedurre che il parallelepipedo che massimizza l’area è un cubo, e che il suo late equivale a \dfrac{\sqrt{3}}{3}d (vedi 19). Quindi vale la disuguaglianza

    \[ 	A(a,b,c)\leq A\left(\frac{\sqrt{3}}{3}d, \frac{\sqrt{3}}{3}d, \frac{\sqrt{3}}{3}d\right)=2d^2. 	\]

    \[\quad\]

    \[\quad\]

Figura 19: un parallelepipedo ed un cubo aventi diagonale uguale.

    \[\quad\]

    \[\quad\]

Pertanto ricordando l’equazione (3) otteniamo la 4:

    \[ 	2ab+2ac+2bc=A(a,b,c)\leq 2d^2=2(a^2+b^2+c^2).  	\]


 

Esercizio 54  (\bigstar\bigstar\largewhitestar\largewhitestar\largewhitestar). Calcolare l’area del triangolo di vertici A(2,3), B(-1,2), C(2,1).

    \[\quad\]

  1. \dfrac{17}{5};
  2.  

  3. 3;
  4.  

  5. \dfrac{2}{3};
  6.  

  7. -3;
  8.  

  9. 1.

Svolgimento.

Risposta 1.

Disegniamo il triangolo in un sistema di assi cartesiani Oxy:

    \[\quad\]

    \[\quad\]

Figura 20: triangolo di vertici A, B e C.

    \[\quad\]

    \[\quad\]

Notiamo che A e C hanno stessa ascissa, quindi AC è parallelo all’asse delle ordinate e quindi l’altezza BH del triangolo \triangle ABC è parallela all’asse delle ascisse. Quindi la base AC del triangolo è la differenza in valore assoluto \vert 3-1\vert =2 delle ordinate di A e C, mentre l’altezza del triangolo è la differenza in valore assoluto delle ascisse di B ed uno qualunque dei punti A e C, ovvero \vert -1-2\vert=3. Pertanto l’area del triangolo è

    \[ 	\frac{\text{base} \cdot \text{altezza}}{2}=\frac{2\cdot 3 }{2}=3.  	\]

La risposta esatta è quindi la 2.


 

Esercizio 55  (\bigstar\bigstar\largewhitestar\largewhitestar\largewhitestar). Il centro della circonferenza di equazione x^2+y^2-2x+3y=1 è:

    \[\quad\]

  1. \left(1,-\dfrac{3}{2}\right);
  2.  

  3. (0,0);
  4.  

  5. (0,-1);
  6.  

  7. (1,2);
  8.  

  9. (2,1);

Svolgimento.

Risposta 1.

L’equazione di una circonferenza di raggio r>0 e centro C(x_0,y_0) è

    \[ 	(x-x_0)^2+(y-y_0)^2=r^2 \implies x^2+y^2-2xx_0-2yy_0=r^2-(x_0^2+y_0^2). 	\]

Nel nostro caso dobbiamo quindi imporre

    \[ 	-2x_0=-2 \qquad \makebox{e} \qquad -2y_0=3 \implies (x_0,y_0)=\left(1, -\frac{3}{2}\right). 	\]

    \[\quad\]

    \[\quad\]

Figura 21: circonferenza centrata in C\left(1,-\dfrac{3}{2}\right).

    \[\quad\]

    \[\quad\]

La risposta corretta è quindi la 1.


 

Esercizio 56  (\bigstar\largewhitestar\largewhitestar\largewhitestar\largewhitestar).

  1. \sqrt{2+x}-\sqrt{x-3};
  2.  

  3. \sqrt{2+x}+\sqrt{x-3};
  4.  

  5. \sqrt{x^2+x-6};
  6.  

  7. \sqrt[4]{(2+x)^2(x-3)^2}.
  8.  

  9. \sqrt{x^2-x-6}.

Svolgimento.

Risposta 5.

Il radicale si può riarrangiare come

    \[ 	\sqrt{(2+x)(x-3)}=\sqrt{2x-6+x^2-3x}=\sqrt{x^2-x-6}. 	\]


 

Esercizio 57  (\bigstar\largewhitestar\largewhitestar\largewhitestar\largewhitestar). Quale dei seguenti numeri differisce dagli altri?

    \[\quad\]

  1. \log_3(27);
  2.  

  3. \ln\left(\dfrac{1}{e}\cdot e^{4}\right);
  4.  

  5. \sqrt{\log_4{\left(2^{18}\right)}};
  6.  

  7. -\log_{\frac{1}{3}}(3^3);
  8.  

  9. \sqrt[3]{\log_2(2\cdot 4^{4})}.

Svolgimento.

Risposta 5.

Tutti i numeri, tranne il 5 sono uguali a 3:

    \[\begin{aligned} 		\log_3(27) &=\log_3(3^3)=3\log_33=3 \\ 		\ln(\frac{1}{e}\cdot e^{4}) &= \ln(e^{-1}\cdot e^4)=\ln(e^{4-1})=3\ln(e)=3; \\ 		\sqrt{\log_4{\left(2^{18}\right)}}& = \sqrt{\log_4{\left(4^{9}\right)}}=\sqrt{9\log_4{4}}=3;\\ 		-\log_{\frac{1}{3}}(3^3) & 		=-\log_{\frac{1}{3}}\left(\left(\frac{1}{3}\right)^{-3}\right)=-(-3)\log_{\frac{1}{3}}\left(\frac{1}{3}\right)=3.\\	 	\end{aligned}\]

Invece il 5 è uguale a

    \[ 	\sqrt[3]{\log_2(2\cdot 4^{4})}=\sqrt[3]{\log_2(2^9)}=\sqrt[3]{9\log_2(2)}=\sqrt[3]{9}\neq 3, 	\]

che è quindi la risposta corretta.


 

Esercizio 58  (\bigstar\bigstar\bigstar\largewhitestar\largewhitestar). Fissato un sistema di riferimento ortogonale Oxy, siano \mathcal C_1 e \mathcal C_2 due circonferenze tangenti, una di centro C_1(-1,1) e di raggio r_1=2 e l’altra di centro C_2 e raggio r_2=3. Quali dei seguenti punti può essere il centro C_2 di \mathcal C_2?

    \[\quad\]

  1. (1,-1);
  2.  

  3. (2,3);
  4.  

  5. (0,0);
  6.  

  7. \left(\sqrt{\dfrac{23}{2}}, \sqrt{\dfrac{23}{2}}\right);
  8.  

  9. nessuna delle precedenti.

Svolgimento.

Risposta 4.

Sia d la distanza fra i centri C_1 e C_2. Le due circonferenze \mathcal C_1 e \mathcal C_2 (vedi figura 22)

    \[\quad\]

  • si intersecano in due punti distinti se e solo se d<r_1+r_2;
  •  

  • sono tangenti se e solo se d=r_1+r_2;
  •  

  • non si intersecano se e solo se d>r_1+r_2.

Nel nostro caso l’unico punto per cui d è uguale a r_1+r_2=2+3=5 è 4. Infatti

    \[ 	d^2=\left(\sqrt{\frac{23}{2}}+1\right)^2+\left(\sqrt{\frac{23}{2}}-1\right)^2=\frac{23}{2}+2\sqrt{\frac{23}{2}}+1+\frac{23}{2}-2\sqrt{\frac{23}{2}}+1=23+1+1=5^2 \implies d=5.  	\]

Nei restanti casi, se il centro C_2 fosse 1 o 3 allora le circonferenze si incontrerebbero in due punti distinti, mentre nel restante caso 2 le circonferenze non si intersecherebbero.

    \[\quad\]

    \[\quad\]

Figura 22: le tre possibili posizioni reciproche di una coppia di circonferenze.

    \[\quad\]

    \[\quad\]

La risposta corretta è quindi la 4.


 

Esercizio 59  (\bigstar\bigstar\bigstar\largewhitestar\largewhitestar). Da un mazzo di 40 carte, composto da 10 cuori, 10 quadri, 10 fiori e 10 picche, se ne estraggono 3; qual è la probabilità di trovare tre carte di semi diversi, supponendo di non rimettere la carta estratta nel mazzo?

    \[\quad\]

  1. \dfrac{400}{246};
  2.  

  3. \dfrac{129}{262};
  4.  

  5. \dfrac{3}{8};
  6.  

  7. \dfrac{6}{7};
  8.  

  9. \dfrac{100}{247}.

Svolgimento.

Risposta 5.

Dobbiamo considerare la probabilità di estrarre un determinato seme ad ogni estrazione:

    \[\quad\]

  1. Alla prima estrazione, la probabilità di estrarre un seme è 1;
  2.  

  3. Alla seconda estrazione, dobbiamo evitare il seme della prima carta. Quindi la probabilità è \dfrac{30}{39}, ovvero 30 carte disponbili diverse dal primo seme su 39 carte rimaste;
  4.  

  5. Alla terza estrazione, dobbiamo evitare sia il primo che il secondo seme. Quindi la probabilità è \dfrac{20}{38}, ovvero 20 carte disponibili su 38 carte rimaste.
  6. La probabilità finale è quindi:

        \[ 	P(3 \text{\ semi \ diversi})=1\cdot \frac{30}{39}\cdot \frac{20}{38}=\frac{100}{247}.  	\]

    La risposta corretta è quindi la 5.


 

Esercizio 60  (\bigstar\bigstar\largewhitestar\largewhitestar\largewhitestar). Dati i seguenti monomi:

    \[ 		7xy^2z^3, \quad 28x^2yz, \qquad 35x^2y^2z^2, 		\]

il loro minimo comune multiplo è:

    \[\quad\]

  1. 7xyz;
  2.  

  3. 140x^3y^2z^3;
  4.  

  5. 140xyz^3;
  6.  

  7. 14xy^2z^3;
  8.  

  9. 140x^2y^2z^3.

Svolgimento.

Risposta 5.

Per calcolare il loro minimo comune multiplo occorre prendere ogni parte letterale ed elevarla al massimo esponente e poi, se i coefficienti dei monomi sono interi come in questo caso, occorre fare il minimo comunque multiplo tra loro. Il monomio che stiamo cercando ha quindi parte letterale uguale a x^2y^2z^3, mentre il coefficiente è

    \[ 	\text{mcm}(7,28,35)=140. 	\]

Perciò il minimo comune multiplo è 140x^2y^2z^3.


 

Esercizio 61  (\bigstar\bigstar\bigstar\largewhitestar\largewhitestar). Quale dei seguenti valori riportati è la migliore approssimazione della radice quadrata di 1265894

    \[\quad\]

  1. 900;
  2.  

  3. 1125;
  4.  

  5. 12001;
  6.  

  7. 421964;
  8.  

  9. 632947.

Svolgimento.

Risposta 2.

La 4 e la 5 sono numeri troppo grandi perché al quadrato facciano 1265894.

Invece:

    \[\quad\]

  • 900^2=81\cdot 10^4=81000;
  •  

  • dato che 1125\approx  11\cdot 100 allora 1125^2\approx 11^2\cdot 10^4=1210000;
  •  

  • dato che 12001\approx 12000 allora 12000^2\approx 144\cdot 10^6=144000000.

Il numero il cui quadrato si avvicina di più a 1265894 è 1125, quindi la risposta corretta è la 2.


 

Esercizio 62  (\bigstar\largewhitestar\largewhitestar\largewhitestar\largewhitestar). Il prezzo di un elettrodomestico è aumentato del 17\% e viene venduto attualmente 234 euro. Qual è il prezzo inziale di vendita?

    \[\quad\]

  1. 156 euro;
  2.  

  3. 217 euro;
  4.  

  5. 200 euro;
  6.  

  7. 100 euro;
  8.  

  9. 117 euro.

Svolgimento.

Risposta 3.

Sia x il prezzo iniziale, allora

    \[ 	234=x+x\frac{17}{100}=\frac{117}{100}x \iff x=\frac{234\cdot 100}{117}=200.  	\]


 

Esercizio 63  (\bigstar\largewhitestar\largewhitestar\largewhitestar\largewhitestar). Il radicale

    \[ 		\sqrt[5]{\frac{7^3\cdot 7^{13}\cdot7^{-5}+ (7^2)^{7}}{7^4+7}} 		\]

è uguale a:

    \[\quad\]

  1. \sqrt[5]{7^2};
  2.  

  3. 7;
  4.  

  5. 49;
  6.  

  7. 11;
  8.  

  9. nessuna delle precedenti.

Svolgimento.

Risposta 3.

Il radicale si può riscrivere come

    \[ 	\sqrt[5]{\frac{7^3\cdot 7^{13}\cdot7^{-5}+ (7^2)^{7}}{7^4+7}}=\sqrt[5]{\frac{ 7^{11}(1+7^3)}{7\cdot \left(7^3+1\right)}}=\sqrt[5]{7^{10}}=49. 	\]


 

Esercizio 64  (\bigstar\bigstar\largewhitestar\largewhitestar\largewhitestar). Quante terne si possono formare con 90 numeri differenti?

    \[\quad\]

  1. 117480;
  2.  

  3. 370822;
  4.  

  5. 4005;
  6.  

  7. 97260;
  8.  

  9. 423651.

Svolgimento.

Risposta 1.

Il numero di terne possibili avendo a disposizione 90 numeri differenti è il numero di modi di selezionare 3 numeri su 90 senza considerare il loro ordine, ovvero il numero di combinazioni di 3 numeri su 90:

    \[ 	\binom{90}{3}=\frac{90\cdot 89\cdot 88\cdot 87!}{(90-3)!\cdot 3!}=\frac{90\cdot 89\cdot 88}{3\cdot 2}=15\cdot 89\cdot 88=117480. 	\]


 

Esercizio 65  (\bigstar\bigstar\largewhitestar\largewhitestar\largewhitestar). Selezionare il corretto ordinamento fra i numeri 3^{12}, 122^3, 2^{15}

    \[\quad\]

  1. 3^{12}<122^3<2^{15};
  2.  

  3. 122^3<3^{12}<2^{15};
  4.  

  5. 122^3<2^{15}<3^{12};
  6.  

  7. 2^{15}<122^3<3^{12};
  8.  

  9. 2^{15}<3^{12}<122^3.

Svolgimento.

Risposta 5.

Partiamo dai primi due numeri 3^{12} e 122^3:

    \[ 	3^{12}\geq 122^3\iff \log_3(3^{12}) \geq \log_3(122^3)\implies 12\geq 3\cdot\log_3(122) >\log_3(3^4)=3\cdot 4=12, 	\]

che è falso. Pertanto 3^{12}< 122^3.

Invece, dato che \log_2(3)\geq \frac{3}{2}, allora

    \[ 	3^{12}\leq 2^{15} \iff \log_2(3^{12})\leq \log_2(2^{15})\implies 18=12\cdot \frac{3}{2}\leq 12\cdot \log_2(3)\leq 15, 	\]

che è falso, quindi 3^{12}>2^{15}. L’ordinamento corretto è quindi 2^{15}<3^{12}<122^3.

La disuguaglianza \log_2(3)\geq \frac{3}{2} si ricava come segue:

    \[ 	\log_2(3)>\frac{3}{2} \iff 3=2^{\log_2(3)}>2^{\frac{3}{2}}=2\sqrt{2} \iff 9=3^2>2^2\cdot 2=8. 	\]


 

Esercizio 66  (\bigstar\bigstar\largewhitestar\largewhitestar\largewhitestar). Sia x un numero reale tale che \dfrac{5}{6}\leq x\leq \dfrac{7}{3}. Allora

    \[\quad\]

  1. \sin(x)< \cos^2(x) <\vert \cos(x)\vert;
  2.  

  3. \cos^2(x)< \sin(x)< \vert \cos(x)\vert;
  4.  

  5. \vert \cos(x)\vert<\cos^2(x) <\sin(x);
  6.  

  7. \cos^2(x)\leq \vert \cos(x)\vert <\sin(x);
  8.  

  9. \vert \cos(x)\vert<\sin(x)<\cos^2(x).

Svolgimento.

Risposta 4.

Dato che \vert \cos(x)\vert è minore di 1, allora il suo quadrato \cos^2(x) è minore o uguale di \vert \cos(x)\vert.

Inoltre osserviamo che \dfrac{5}{6}> \dfrac{\pi}{4} e \dfrac{7}{3}< \dfrac{3}{4}\pi, perciò \dfrac{\pi}{4}< x< \dfrac{3}{4}\pi.

In questo intervallo \sin(x) ha misura positiva e osservando la figura 23 si nota che la lunghezza \vert \cos(x)\vert della proiezione sull’asse x è sempre minore di quella sull’asse y, che è \vert \sin(x)\vert =\sin(x). Quindi \vert \cos(x)\vert< \sin(x) e la risposta corretta è la 4.

    \[\quad\]

    \[\quad\]

Figura 23: la circonferenza goniometrica e il settore circolare dei punti per cui \dfrac{5}{6}\leq x\leq \dfrac{7}{3}.

    \[\quad\]

    \[\quad\]


 

Esercizio 67  (\bigstar\bigstar\bigstar\largewhitestar\largewhitestar). L’equazione dell’iperbole che ha due fuochi in (-2,0) e (2,0) e di eccentricità e=2 è

    \[\quad\]

  1. x^2+\dfrac{y^2}{7}=1;
  2.  

  3. x^2-y^2=1;
  4.  

  5. x^2-\dfrac{y^2}{3}=1;
  6.  

  7. \dfrac{x^2}{2}-\dfrac{y^2}{3}=1;
  8.  

  9. nessuna delle precedenti.

Svolgimento.

Risposta 3.

L’iperbole è il luogo dei punti P(x,y) del piano Oxy per cui la differenza (in valore assoluto) tra le distanze tra due fuochi F_1 e F_2 è costante:

    \[ 	\vert \| PF_1\|-\|PF_2\|\vert =k=cost. 	\]

Nel caso di due fuochi F_1(-c,0) e F_2(c,0), c\geq 0, simmetrici rispetto l’asse y abbiamo che

    \[ 	\|PF_1\|^2=(x+c)^2+y^2 \qquad \makebox{e} \qquad \|PF_2\|^2=(x-c)^2+y^2, 	\]

da cui si ottiene

    \[ 	\begin{split} 		\left(\|PF_1\|-\|PF_2\|\right)^2= k^2=2x^2+2c^2+2y^2-2\sqrt{\left((x-c)^2+y^2\right)\cdot \left((x+c)^2+y^2\right)} 	\end{split} 	\]

quindi

    \[ 	\left(x^2+c^2+y^2-\dfrac{k^2}{2}\right)^2=\left((x-c)^2+y^2\right)\cdot \left((x+c)^2+y^2\right)=(x^2-c^2)^2+2y^2(x^2+c^2)+y^4. 	\]

Facendo le opportune semplificazioni si arriva a

    \[ 	\dfrac{k^4}{4}+4x^2c^2-x^2k^2-c^2k^2-y^2k^2=0 \implies \dfrac{x^2}{\dfrac{k^2}{4}}-\dfrac{y^2}{c^2-\dfrac{k^2}{4}}=1.  	\]

Perciò l’equazione dell’iperbole è del tipo \dfrac{x^2}{a^2}-\dfrac{y^2}{b^2}=1 e la relazione fra i coefficienti a, b e quelli c, k è la seguente:

    \[ 	\begin{cases} 		a^2=\dfrac{k^2}{4} \\ 		c^2=a^2+b^2 	\end{cases}. 	\]

Per definizione l’eccentricità di una iperbole è e=\dfrac{c}{a}, perciò occorre risolvere il sistema

    \[ 	\begin{cases} 		2=\dfrac{2}{a} \\ 		2^2=a^2+b^2 	\end{cases} \implies \begin{cases} 		a^2=1 \\ 		b^2=3 	\end{cases}. 	\]

    \[\quad\]

    \[\quad\]

Figura 24: iperbole di fuochi F_1=(-2,0) e F_2=(2,0) ed eccentricità e=2.

    \[\quad\]

    \[\quad\]

Perciò l’equazione dell’iperbole è x^2-\dfrac{y^2}{3}=1.

Un altro modo più veloce per determinare l’equazione dell’iperbole sarebbe potuto essere partire dall’equazione canonica \dfrac{x^2}{a^2}-\dfrac{y^2}{b^2}=1 e ricordare che valgono le relazioni c=\sqrt{a^2+b^2} e e=\dfrac{c}{a}.


 

Esercizio 68  (\bigstar\bigstar\bigstar\largewhitestar\largewhitestar). In una catena di montaggio vengono prodotte batterie per telefoni cellulari. Ogni telefono per funzionare deve contenere 4 batterie e se una sola di tra loro non funziona, allora il telefono non si accende.

Le batterie sono state scelte a caso tra le 1000 prodotte, di cui 98 sono guaste. Qual è la probabilità che il telefono cellulare funzioni?

    \[\quad\]

  1. 0,5;
  2.  

  3. circa 0,66;
  4.  

  5. circa 0,89;
  6.  

  7. circa 0,75;
  8.  

  9. circa 0,12.

Svolgimento.

Risposta 2.

La probabilità che la prima delle 4 batterie funzioni è il numero di batterie funzionanti, ovvero 902, sulle 1000 disponbiili. La probabilità che la seconda batteria funzioni è il numero delle restanti batterie funzionanti, ovvero 901, su 999 batterie disponibili. Continuando a ragionare così, la probabilità che la terza batteria funzioni è 900 su 998, e che la quarta funzioni è 899 su 997. Quindi la probabilità che il telefono si accenda è

    \[ 	\frac{902}{1000}\cdot \frac{901}{999}\cdot \frac{900}{998}\cdot \frac{899}{997}=\frac{451}{5}\cdot  \frac{901}{111}\cdot \frac{1}{998} \cdot \frac{899}{997}\approx 0,66. 	\]

La risposta corretta è quindi la 2.


 

Esercizio 69  (\bigstar\bigstar\largewhitestar\largewhitestar\largewhitestar). Su di un segmento AB lungo 14 cm si sceglie un punto interno P in modo che l’area dei figura piana definita dai due quadrati, costruiti dalla stessa parte rispetto la retta AB e aventi lati AP e PB, sia uguale a 100 \text{cm}^2. Il perimetro della figura è:

    \[\quad\]

  1. 12 cm;
  2.  

  3. 41 cm;
  4.  

  5. 32 cm;
  6.  

  7. 44 cm;
  8.  

  9. 27 cm.

Svolgimento.

Risposta 4.

Indichiamo con a la lunghezza di AP e con b la lunghezza di PB. A meno di scambiare a con b possiamo supporre che b\geq a. Dai dati abbiamo che

    \[ 	\begin{cases} 		a+b=14 \\ 		a^2+b^2=100 	\end{cases} \implies 14^2=(a+b)^2=a^2+2ab+b^2=100+2ab  \implies ab=48.  	\]

Allora a e b sono soluzioni dell’equazione di secondo grado x^2-(a+b)x+ab=x^2-14x+48=0:

    \[ 	x_{1,2}=\frac{14\pm \sqrt{14^2-4\cdot 48}}{2}=\frac{14\pm 2}{2} \implies \quad  a=6 \qquad \makebox{e} \qquad b=8.  	\]

Dunque il perimetro della Figura ?? è

    \[ 	a+a+a+(b-a)+b+b+b=2a+4b=2(a+b)+2b=28+16=44  \ \text{cm}. 	\]

    \[\quad\]

    \[\quad\]

Figura 25: i due quadrati costruiti sul segmento AB e l’area della figura.

    \[\quad\]

    \[\quad\]

La risposta corretta è quindi la 4.


 

Esercizio 70  (\bigstar\bigstar\bigstar\largewhitestar\largewhitestar). Si hanno 3 monete, una delle quali è truccata e la sua probabilità che esca testa è del 75\%. Qual è la probabilità che esca due volte croce e una testa?

    \[\quad\]

  1. circa 0,16;
  2.  

  3. 0,56;
  4.  

  5. \dfrac{5}{16};
  6.  

  7. 2;
  8.  

  9. 0,75:

Svolgimento.

Risposta 3.

Possiamo supporre che la prima moneta sia quella truccata. In questo caso, dato che la probabilità che esca un risultato qualunque dalla prima moneta è 1, se la probabilità che esca testa è del 75 \%, allora quella che esca croce deve essere del 25\%, cioè rispettivamente \dfrac{3}{4} ed \dfrac{1}{4}. Allora

    \[ 	\begin{split} 		P(testa, croce, croce) & =\frac{3}{4}\cdot \frac{1}{2}\cdot \frac{1}{2}=\frac{3}{16},\\ 		P(croce, testa, croce) &=\frac{1}{4}\cdot \frac{1}{2}\cdot \frac{1}{2}=\frac{1}{16}, \\ 		P(croce, croce, testa) & =\frac{1}{4}\cdot \frac{1}{2}\cdot \frac{1}{2}=\frac{1}{16}. 	\end{split} 	\]

Perciò la probabilità che esca due volte croce e una testa è

    \[ 	\frac{3}{16}+\frac{1}{16}+\frac{1}{16}=\frac{5}{16}.  	\]


 

Esercizio 71  (\bigstar\bigstar\largewhitestar\largewhitestar\largewhitestar). Il rapporto tra l’area di un cerchio e quella di un quadrato circoscritto ad esso è

    \[\quad\]

  1. \dfrac{2}{5}\pi;
  2.  

  3. \dfrac{1}{4};
  4.  

  5. \dfrac{1}{3};
  6.  

  7. maggiore di 1;
  8.  

  9. \dfrac{\pi}{4}.

Svolgimento.

Risposta 5.

Il lato del quadrato è il doppio del raggio r del cerchio, quindi l’area del quadrato è (2r)^2=4r^2. D’altro canto, l’area del cerchio è \pi r^2, perciò il loro rapporto è

    \[ 	\frac{\pi r^2}{4r^2}=\frac{\pi}{4}.  	\]

    \[\quad\]

    \[\quad\]

Figura 26: il quadrato circoscritto nel cerchio.

    \[\quad\]

    \[\quad\]


 

Esercizio 72  (\bigstar\bigstar\bigstar\largewhitestar\largewhitestar). La somma delle cotangenti trigonometriche degli angoli interni di un dodecagono regolare è

    \[\quad\]

  1. -12\sqrt{3};
  2.  

  3. 4\sqrt{3};
  4.  

  5. -4\sqrt{3};
  6.  

  7. \sqrt{3};
  8.  

  9. \sqrt{12}.

Svolgimento.

Risposta 1.

Dividendo l’angolo giro 2\pi per 12 otteniamo l’angolo interno di uno dei triangoli che compongono il dodecagono (vedi figura 27). Dato che il triangolo è isoscele, allora i restanti due angoli interni al triangolo misurano entrambi \dfrac{1}{2}\left(\pi- \dfrac{\pi}{6}\right)=\dfrac{5}{12}\pi. Perciò uno degli angoli interni del dodecagono è 2\cdot \dfrac{5}{12}\pi=\dfrac{5}{6}\pi.

    \[\quad\]

    \[\quad\]

Figura 27: dodecagono regolare.

    \[\quad\]

    \[\quad\]

Quindi la somma delle cotagenti è

    \[ 	12\cdot \cot\left(\frac{5}{6}\pi\right)=-12\sqrt{3}, 	\]

e la risposta corretta è la 1.


 

Esercizio 73  (\bigstar\bigstar\largewhitestar\largewhitestar\largewhitestar). Una torta di forma cilindrica viene tagliata in 17 fette tutte uguali tra loro. Se il diametro della torta è di 34 cm, quanto vale il rapporto tra il volume di ciascuna fetta e l’altezza del cilindro?

    \[\quad\]

  1. 34\pi;
  2.  

  3. 17\pi;
  4.  

  5. 1;
  6.  

  7. 12\pi;
  8.  

  9. 9\pi.

Svolgimento.

Risposta 2.

Il volume di un cilindro è \pi hr^2 dove r è il raggio della base del cilindro ed h è la sua altezza. Il volume di una fetta è allora \dfrac{\pi}{17}h r^2=\dfrac{\pi}{4\cdot 17}h (2r)^2=\dfrac{\pi}{17\cdot 4}h(34)^2=17 h\pi e quindi il rapporto con l’altezza della torta è 17\pi.

    \[\quad\]

    \[\quad\]

Figura 28: la torta a forma cilindrica tagliata in 17 fette.

    \[\quad\]

    \[\quad\]

Quindi la riposta corretta è la 2.


 

Esercizio 74  (\bigstar\bigstar\largewhitestar\largewhitestar\largewhitestar). Quali sono le soluzioni reali della seguente disequazione?

    \[ 		\log^2(x)+\log(x^2)-8\geq 0. 		\]

    \[\quad\]

  1. x=\log(2);
  2.  

  3. x\leq \dfrac{1}{e^4}, x\geq e^2;
  4.  

  5. 0< x\leq \dfrac{1}{e^3}, x\geq e^3;
  6.  

  7. x\leq \dfrac{1}{e^3}, x\geq e^3;
  8.  

  9. 0<x\leq \dfrac{1}{e^4}, x\geq e^2.

Svolgimento.

Risposta 5.

La disequazione data ha senso solo per x>0, che è dove il logaritmo esiste.

Osserviamo che \log(x^2)=2\log(x), per cui ponendo z:=\log(x) si ottiene

    \[ 	z^2+2z-8\geq 0. 	\]

Le soluzioni dell’equazione z^2+2z-8=0 sono

    \[ 	z_{1,2}=\frac{-1\pm \sqrt{1^2-(-8)}}{1}= -1\pm 3 \quad \implies \quad  z_1=-4 \qquad \makebox{e} \qquad z_2=2.  	\]

I valori di z per cui z^2+2z-8 è non negativo sono quindi z\leq -4, z\geq  2. Adesso sostituiamo di nuovo \log(x) al posto di z e risolviamo le due diesquazioni:

    \[ 	\log(x)\leq -4\implies x\leq e^{-4} \qquad \makebox{e} \qquad \log(x)\geq 2 \implies x\geq e^2. 	\]

quindi 0< x\leq \dfrac{1}{e^4}, x\geq  e^2.


 

Esercizio 75  (\bigstar\largewhitestar\largewhitestar\largewhitestar\largewhitestar). Il prodotto di due numeri irrazionali distinti

    \[\quad\]

  1. è un numero immaginario;
  2.  

  3. è sempre irrazionale;
  4.  

  5. può essere razionale;
  6.  

  7. non è mai razionale;
  8.  

  9. è sempre positivo.

Svolgimento.

Risposta 3.

Il prodotto di numeri irrazionali distinti può essere razionale, ad esempio \sqrt{2}\cdot \sqrt{8}=4, che è razionale.


 

Esercizio 76  (\bigstar\largewhitestar\largewhitestar\largewhitestar\largewhitestar). Se x è un numero reale negativo, allora:

    \[\quad\]

  1. x+\vert x \vert=0;
  2.  

  3. x-\vert x\vert >0;
  4.  

  5. x\cdot \vert x\vert >0;
  6.  

  7. \vert x\vert < 0;
  8.  

  9. -x\cdot \vert x\vert<0;

Svolgimento.

Risposta 1.

Visto che x è un numero negativo, allora \vert x\vert=-x, perciò x+\vert x\vert =x-x=0.


 

Esercizio 77  (\bigstar\bigstar\bigstar\bigstar\largewhitestar). Dato un cono di altezza 2R e raggio R, la quantità (non nulla) di cui biosogna diminuire l’altezza e aumentare il raggio per cui l’area di superficie totale non cambi

    \[\quad\]

  1. è 3R;
  2.  

  3. è R;
  4.  

  5. è \dfrac{1+\sqrt{5}}{2}R;
  6.  

  7. è soluzione di un’equazione polinomiale di 3 grado;
  8.  

  9. non esiste.

Svolgimento.

Risposta 5.

L’area della superficie totale del cono è A_{tot}=\pi R(R+a), dove a è l’apotema del cono. In particolare, valore dell’apotema nel nostro caso è a=\sqrt{(2R)^2+R^2})=\sqrt{5}R, quindi la superficie totale del cono si può riscrivere come

    \[ 	A_{tot}=\pi R(R+a)=\pi R^2(1+\sqrt{5}). 	\]

Indichiamo con x>0 la quantità per cui il nuovo cono di altezza 2R-x e raggio R+x ha sempre area di superifice totale uguale a \pi R^2(1+\sqrt{5}). Allora x soddisfa l’equazione

    \[ 	A_{tot \ nuovo \ cono}=\pi \left(R+x\right)\left(R+x+\sqrt{(2R-x)^2+(R+x)^2}\right)=\pi R^2(1+\sqrt{5}).  	\]

Risolviamo l’equazione ottenuta:

    \[ 	\begin{split} 		\sqrt{(2R-x)^2+(R+x)^2} & =\frac{R^2(1+\sqrt{5})}{R+x}-(R+x) \iff \\ 		\left((2R-x)^2+(R+x)^2 \right)(R+x)^2 & =\left(R^2(1+\sqrt{5})-(R+x)^2\right)^2 \iff \\ 		(2R-x)^2(R+x)^2 & =R^4(1+\sqrt{5})^2-2(R+x)^2R^2(1+\sqrt{5}). \\ 	\end{split} 	\]

Continuando a semplificare otteniamo

    \[ 	\begin{split} 		(4R^2+x^2-4Rx+2R^2(1+\sqrt{5}))(R+x)^2 & =R^4(1+\sqrt{5})^2 \iff \\ 		x^4+R^2x^2+2Rx^3-4Rx^3-4R^3x-8Rx^2+2R^2(3+\sqrt{5})(R^2+x^2+2Rx) & =R^4(1+\sqrt{5})^2 \iff  \\ 		x^4-2Rx^3+ \left(R^2-8R^2+2R^2(3+\sqrt{5})\right)x^2+  \left(-4R^3+4R^3(3+\sqrt{5})\right)x  & =0 \iff  \\ 		x^4-2Rx^3+R^2\left(-1+2\sqrt{5}\right)x^2+ 4R^3\left(2+\sqrt{5}\right)x  & =0. 	\end{split} 	\]

Come ci si poteva aspettare, una delle soluzioni è proprio x=0, per cui escludendo questa soluzione e dividendo l’equazione di sopra per x otteniamo che altre possibili soluzioni devono essere radici del polinomio di terzo grado

    \[ 	x^3-2Rx^2+R^2\left(-1+2\sqrt{5}\right)x+4R^3\left(2+\sqrt{5}\right) =0. 	\]

Questo polinomio ha due soluzioni complesse ed una sola soluzione reale negativa, che è circa -\dfrac{17}{10}R. Dato che x deve essere positivo, allora non esiste alcuna soluzione al problema.

    \[\quad\]

    \[\quad\]

Figura 29: il cono di altezza 2R e raggio R a sinistra, e quello di altezza 2R-x e base R+x, a destra.

    \[\quad\]

    \[\quad\]


 

Esercizio 78  (\bigstar\bigstar\bigstar\largewhitestar\largewhitestar). Una clessidra è costituita dall’unione di due coni circolari uguali. Dopo 13 minuti il livello della sabbia nella parte superiore è sceso di \dfrac{2}{3} dell’altezza. Assumendo che il flusso della sabbia resti costante nel tempo, quanto manca affinchè la clessidra si svuoti completamente?

    \[\quad\]

  1. 5 minuti;
  2.  

  3. \dfrac{13}{3} minuti;
  4.  

  5. 14 minuti;
  6.  

  7. 30 secondi;
  8.  

  9. 1 minuto.

Svolgimento.

Risposta 4.

Siano H l’altezza del cono superiore e h l’altezza del cono dopo 13 minuti, quindi h=\dfrac{1}{3}H. Siano R ed r i raggi del cono superiore all’altezza H e h rispettivamente, e V_1, V_2 i volumi rispettivi dei due coni.

La quantità di sabbia uscita dal cono superiore dopo 13 minuti è il volume V_1-V_2 del tronco di cono, quindi per determinare il tempo rimanente t affinché la clessidra si svuoti completamente si può usare la proporzione:

(4)   \begin{equation*} 		\frac{V_2}{V_1-V_2}=\frac{t}{13} \implies t=13\cdot \frac{1}{\dfrac{V_1}{V_2}-1}. 	\end{equation*}

Quindi occorre calcolare il rapporto dei volumi dei due coni.

I due triangoli in figura 30 sono simili tra loro, perciò si deduce che

    \[ 	\frac{r}{R}=\frac{h}{H}=\frac{1}{3}.  	\]

Quindi

    \[ 	V_1=\frac{\pi R^2 H}{3}, \qquad V_2=\frac{\pi r^2 h}{3} \implies \frac{V_1}{V_2}=\frac{R^3}{r^3}=3^3=27. 	\]

Sostituendo in (4) il valore del rapporto dei due volumi, si ottiene che il tempo rimanente è di 30 secondi.

    \[\quad\]

    \[\quad\]

Figura 30: cono circolare, che costiuisce la parte superiore della clessidra.

    \[\quad\]

    \[\quad\]

La risposta corretta è quindi la 4.


 

Esercizio 79  (\bigstar\bigstar\largewhitestar\largewhitestar\largewhitestar). Quante volte occorre lanciare una moneta per essere sicuri che sia uscito almeno due volte testa o due volte croce?

    \[\quad\]

  1. tre;
  2.  

  3. una;
  4.  

  5. due;
  6.  

  7. quattro;
  8.  

  9. non si può stabilire.

Svolgimento.

Risposta 1.

La moneta deve essere lanciata almeno tre volte. Se fosse stata lanciata esattamente due volte, potrebbe accadere che sia uscito testa e croce, quindi due lanci non bastano. Invece, al terzo lancio può uscire solamente o testa, o croce, garantendo quindi che su tre lanci sia uscito sempre almeno o due volte testa o due volte croce.


 

Esercizio 80  (\bigstar\bigstar\largewhitestar\largewhitestar\largewhitestar). La soluzione della disequazione \vert x\vert ^5\geq  x^6 è

    \[\quad\]

  1. -1\leq x\leq1;
  2.  

  3. x\geq 1;
  4.  

  5. -1\leq x \leq \dfrac{1}{2};
  6.  

  7. x=1;
  8.  

  9. x\leq 0.

Svolgimento.

Risposta 1.

Dato che x^6 è sempre non negativo, allora \vert x\vert^6=x^6, quindi otteniamo

    \[ 	\vert x\vert ^5\geq x^6 \iff \vert x\vert ^6-\vert x\vert ^5\leq 0 \iff \vert x\vert ^5(\vert x\vert -1)\leq 0. \]

\vert x\vert ^5 è sempre maggiore o uguale a zero, quindi il segno di \vert x\vert ^5(\vert x\vert -1) è quello di \vert x\vert -1, che è minore o uguale di 0 per -1\leq x\leq 1.


 

Esercizio 81  (\bigstar\bigstar\bigstar\largewhitestar\largewhitestar). Dati due quadrilateri di uguali angoli, le lunghezze del quadrilatero più piccolo sono il 75 per cento delle rispettive lunghezze del più grande. Allora il rapporto tra l’area del quadrilatero più grande e quello più piccolo è

    \[\quad\]

  1. \dfrac{16}{9};
  2.  

  3. \dfrac{3}{4};
  4.  

  5. \dfrac{1}{4};
  6.  

  7. \dfrac{1}{16};
  8.  

  9. i dati forniti non sono sufficienti per ottenere questa informazione.

Svolgimento 1.

Risposta 1.

Ci sono diversi metodi per risolvere il problema. Uno può essere quello di osservare che i quadrilateri, avendo angoli uguali e lati in proporzione, sono simili e quindi tracciando due diagonali corrispondenti si decompongono in triangoli simili. I lati e le altezze dei triangoli \triangle EFG e \triangle EHG sono rispettivamente i \dfrac{75}{100}=\dfrac{3}{4} dei corrispettivi triangoli \triangle ABC e \triangle ACD. Quindi, il rapporto delle aree di \triangle EFG e \triangle EHG sono pari a \left(\dfrac{3}{4}\right)^2=\dfrac{9}{16} delle aree rispettivamente di \triangle ABC e \triangle ACD. La risposta corretta è quindi la 1.


Svolgimento. 2

Un altro metodo può essere il seguente, che fa uso della trigonometria.

Indichiamo con ABCD il quadrilatero più grande e con EFGH il secondo, come in figura 31. Allora

    \[ 	\begin{split} 		A_{\triangle{ABC}} & =\frac{1}{2}\overline{AB}\cdot \overline{BC}\cdot \sin(\angle{ABC}) \qquad \makebox{e} \qquad A_{\triangle{CDA}}=\frac{1}{2}\overline{CD}\cdot \overline{DA}\cdot \sin(\angle{CDA}), \\ 		A_{\triangle{EFG}} & =\frac{1}{2}\overline{EF}\cdot \overline{FG}\cdot \sin(\angle{EFG}) \qquad \makebox{e} \qquad A_{\triangle{GHE}}=\frac{1}{2}\overline{GH}\cdot \overline{HE}\cdot \sin(\angle{GHE}). 	\end{split} 	\]

Quindi l’area dei due quadrilateri è

    \[ 	\begin{split} 		A_{ABCD} & =	A_{\triangle{ABC}}+  A_{\triangle{CDA}}=\frac{1}{2}\left(\overline{AB}\cdot \overline{BC}\cdot \sin(\angle{ABC})+\overline{CD}\cdot \overline{DA}\cdot \sin(\angle{CDA})\right), \\ 		A_{EFGH} &= A_{\triangle{EFG}} + A_{\triangle{GHE}}=\frac{1}{2}\left(\overline{EF}\cdot \overline{FG}\cdot \sin(\angle{EFG})+\overline{GH}\cdot \overline{HE}\cdot \sin(\angle{GHE})\right). 	\end{split} 	\]

Adesso usiamo che \angle{ABC}=\angle{EFG}, \angle{CDA}=\angle{GHE}, e che

    \[ 	\overline{EF}=\frac{3}{4}\overline{AB} \qquad \overline{FG}=\frac{3}{4}\overline{BC} \qquad \overline{GH}=\frac{3}{4}\overline{CD} \qquad \overline{HE}=\frac{3}{4}\overline{DA} 	\]

per ottenere

    \[ 	\frac{A_{ABCD}}{A_{EFGH}}=\frac{\overline{AB}\cdot \overline{BC}\cdot \sin(\angle{ABC})+\overline{CD}\cdot \overline{DA}\cdot \sin(\angle{CDA})}{\overline{EF}\cdot \overline{FG}\cdot \sin(\angle{EFG})+\overline{GH}\cdot \overline{HE}\cdot \sin(\angle{GHE})}=\frac{16}{9}. 	\]

    \[\quad\]

    \[\quad\]

Figura 31: i due quadrilateri ABCD e EFGH.

    \[\quad\]

    \[\quad\]

Il risultato ottenuto mostra come il rapporto tra le areee dei due quadrilateri è uguale al rapporto tra i loro lati al quadrato. Questo risultato non è un caso in quanto è sempre vero che se due figure sono simili e il rapporto dei loro lati è k, allora il rapporto delle loro aree è il quadrato k^2 (come anche il rapporto dei loro volumi sarebbe k^3).

La risposta corretta è quindi la 1.


 

Esercizio 82  (\bigstar\bigstar\largewhitestar\largewhitestar\largewhitestar). Dato a>0, a\neq 1, il prodotto \ln(a)\log_a(e) è uguale a

    \[\quad\]

  1. 2;
  2.  

  3. 1;
  4.  

  5. 0;
  6.  

  7. \pi;
  8.  

  9. \dfrac{\log_a(e)}{ln(-a)}.

Svolgimento.

Risposta 2.

Utilizziamo la proprietà dei logaritmi per cui \log (x^y)=y\log(x):

    \[ 	\ln(a)\log_a(e)=\ln(a^{\log_a(e)})=\ln(e)=1. 	\]

La risposta corretta è quindi la 2, plausibile è il 5.


 

Esercizio 83  (\bigstar\bigstar\bigstar\largewhitestar\largewhitestar). L’equazione (x^2+y^2+1)\left(x^2+\dfrac{y^2}{2}-1\right)=0 rappresenta nel piano Oxy

    \[\quad\]

  1. un’iperbole;
  2.  

  3. un’ellisse;
  4.  

  5. un’ellisse ed una circonferenza;
  6.  

  7. un’ellisse ed un punto;
  8.  

  9. nesuna delle precedenti.

Svolgimento.

Risposta 2.

Le coppie (x,y) che soddisfazione l’equazione devono verificare

    \[ 	x^2+y^2+1=0 \qquad \makebox{oppure} \qquad x^2+\frac{y^2}{2}-1=0. 	\]

Tuttavia, la prima equazione non ammettte soluzioni nel campo dei numeri reali, quindi (x,y) deve verificare necessariamente

    \[ 	x^2+\frac{y^2}{2}-1=0 	\]

che è l’equazione di un’ellisse.

    \[\quad\]

    \[\quad\]

Figura 32: l’insieme dei punti che verificano l’equazione (x^2+y^2+1)\left(x^2+\dfrac{y^2}{2}-1\right)=0.

    \[\quad\]

    \[\quad\]

La risposta corretta è quindi la 2.


 

Esercizio 84  (\bigstar\largewhitestar\largewhitestar\largewhitestar\largewhitestar). Il polinomio x^4 - 10x^3 + 35x^2 - 50x + 24 coincide con

    \[\quad\]

  1. (x-1)(x-2)(x-4)(x-5);
  2.  

  3. (x-1)^2(x-4)^2;
  4.  

  5. (2x-1)^3(x-2);
  6.  

  7. (x-2)(x+3)(x+4)^2;
  8.  

  9. (x-1)(x-2)(x-3)(x-4).

Svolgimento.

Risposta 5.

Un approccio diretto è trovare delle radici banali del polinomio e poi usare la regola di Ruffini per scomporlo in fattori, come fatto nell’ esercizio 16. Tuttavia, in questo caso conviene ragionare per esclusione. Infatti, notiamo subito che x=1 è radice del polinomio:

    \[ 	1-10+35-50+24=0. 	\]

Questo esclude subito 3 e 4. Inoltre, il polinomio in x=0 è uguale a 24, quindi l’unico polinomio plausibile è 5.


 

Esercizio 85  (\bigstar\bigstar\bigstar\largewhitestar\largewhitestar). Tra i rombi aventi perimetro fissato, quello di area massima ha

    \[\quad\]

  1. una diagonale che è il doppio dell’altra;
  2.  

  3. una diagonale che è il quadruplo dell’altra;
  4.  

  5. le due diagonali uguali, ovvero è un quadrato;
  6.  

  7. la diagonale maggiore che è \dfrac{3}{2} la diagonale minore;
  8.  

  9. nessuna delle precedenti.

Svolgimento.

Risposta 3.

Sia x la diagonale minore e y la diagonale maggiore del rombo. L’area del rombo è quindi

    \[ 	A(x,y)=\frac{1}{2}xy. 	\]

Invece il perimetro fissato p del rombo è uguale a

    \[ 	p=4\sqrt{x^2+y^2}.  	\]

Osserviamo che

    \[ 	(x-y)^2=x^2+y^2-2xy \implies xy=\frac{1}{2}\left(x^2+y^2-(x-y)^2\right)=\frac{1}{2}\left(\frac{p^2}{16}-(x-y)^2\right). \]

Quindi l’area può essere riscritta come

    \[ 	A(x,y)=\frac{1}{2}xy=\frac{1}{4}\left(\frac{p^2}{16}-(x-y)^2\right). 	\]

Visto che (x-y)^2 è sempre maggiore o uguale a zero, allora A(x,y) è massima per (x-y)^2=0, ovvero per x=y. Abbiamo quindi ottenuto che il rombo deve essere un quadrato.

    \[\quad\]

    \[\quad\]

Figura 33: due rombi di uguale perimetro p. Il quadrato a destra è il rombo di area massima fra quelli di perimetro fissato p.

    \[\quad\]

    \[\quad\]

La risposta corretta è quindi la 3.


 

Esercizio 86  (\bigstar\bigstar\largewhitestar\largewhitestar\largewhitestar). Il valore positivo che rende minima la somma di un numero diverso da 0 con il suo reciproco è

    \[\quad\]

  1. 2
  2.  

  3. \sqrt{2}
  4.  

  5. 1;
  6.  

  7. \dfrac{1}{2}
  8.  

  9. \dfrac{\sqrt{3}}{3}.

Svolgimento.

Risposta 3.

Occorre trovare i punti di minimo della funzione f(x)=x+\dfrac{1}{x}, per x>0. La funzione è derivabile nel suo dominio, quindi i punti di massimo e minimo annullano la derivata, ed il suo segno determina la crescenza e decrescenza della funzione. Nell’intervallo (0,+\infty) abbiamo che

    \[ 	f'(x):=\frac{2x^2-(x^2+1)}{x^2}=\frac{x^2-1}{x^2}\geq 0 \iff  x^2\geq 1 \iff x\geq 1.  	\]

Perciò x=1 è un punto di minimo locale per f, che è anche assoluto in (0,+\infty).

La risposta corretta è quindi la 3.


 

Esercizio 87  (\bigstar\bigstar\largewhitestar\largewhitestar\largewhitestar). La funzione y=\cos^2(2x) ha periodo uguale a

    \[\quad\]

  1. non ha periodo;
  2.  

  3. 3\pi;
  4.  

  5. 2\pi;
  6.  

  7. \pi;
  8.  

  9. \dfrac{\pi}{2}.

Svolgimento.

Risposta 5.

La funzione coseno ha periodo 2\pi, quindi y=\cos(2x) avrà periodo \pi, ed il periodo di y=\cos^2(2x) deve essere quindi minore o uguale a \pi, che esclude 1, 2 e 3. Non è difficile dimostrare che il periodo è \dfrac{\pi}{2}. Verifichiamolo:

    \[ 	\cos^2\left(2\left(x+\frac{\pi}{2}\right)\right)=\cos^2\left(2x+\pi\right)=\left(-\cos\left(2x\right)\right)^2=\cos^2\left(2x\right).  	\]

    \[\quad\]

    \[\quad\]

Figura 34: grafico della funzione y=\cos^2(2x).

    \[\quad\]

    \[\quad\]

La risposta corretta è quindi la 5.


 

Esercizio 88  (\bigstar\bigstar\bigstar\bigstar\largewhitestar). Data una circonferenza di raggio R, fissati due punti A e B sulla circonferenza e un punto Q sulla corda \overline{AB} tale per cui \overline{AQ} e \overline{QB} hanno rispettivamente lunghezza a e b, allora la lunghezza minima tra i segmenti \overline{PQ} dove P è un punto che varia sulla circonferenza è

    \[\quad\]

  1. R-\sqrt{R^2-ab};
  2.  

  3. b;
  4.  

  5. R\cdot \min\{\dfrac{b}{R},\dfrac{a}{R}\};
  6.  

  7. a;
  8.  

  9. nessuna delle precedenti.

Svolgimento.

Risposta 1.

Dimostriamo che il punto P per cui la lunghezza di \overline{PQ} è minima è tale per cui P, Q ed il centro O della circonferenza sono allineati, con P dalla parte di Q. Chiamiamo con P_0 il punto per cui P_0, Q ed O sono allineati, con P_0 dalla parte di Q. Vogliamo quindi mostrare che \vert \vert \overline{P_0Q}\vert 	\vert \leq \vert \vert \overline{PQ}\vert 	\vert per un qualunque punto P sulla circonferenza. Indichiamo con F il punto della circonferenza ottenuto dal prolungamento del segmento \overline{PQ}. Allora la lunghezza di \overline{PQ} è minima se e solo se quella del segmento \overline{QF} è massima, cioè basta dimostrare che \vert \vert \overline{QF}\vert 	\vert \leq \vert \vert \overline{QF_0}\vert 	\vert per ogni punto F della circonferenza (vedi figura 35). Dato che la somma di due lati di un triangolo è sempre maggiore o uguale al terzo lato, allora deduciamo che per \triangle{QOF} vale

    \[ 	\vert \vert \overline{QF}\vert \vert \leq \vert \vert\overline{QO}\vert \vert +\vert \vert \overline{OF}\vert \vert =\vert \vert \overline{QO}\vert \vert +\vert \vert \overline{OF_0}\vert \vert =\vert \vert \overline{QF_0}\vert \vert,  	\]

che è proprio ciò che volevamo dimostrare.

    \[\quad\]

    \[\quad\]

Figura 35: il segmento \overline{QF_0} è quello di lunghezza massima.

    \[\quad\]

    \[\quad\]

Rimane da calcolare la lunghezza di x:=\overline{P_0Q}. Si potrebbe usare uno dei risultati di geometria classica per cui se due corde si intersecano, i segmenti che si formano su una di esse sono i medi e i segmenti dell’altra sono gli estremi di una stessa proporzione.

Presentiamo un metodo alternativo usando un risultato equivalente, il Teorema delle Corde (Proposizione 35 del Libro III di Euclide), per cui se due corde di una circonferenza si intersecano, allora il rettangolo con lati congruenti alle due parti di una corda ha la stessa area del rettangolo con lati congruenti alle due parti dell’altra (vedi figura 36). Possiamo quindi concludere che

    \[ 	x(2R-x)=ab \implies x^2-2Rx+ab=0 \implies x_{1,2}=R\pm \sqrt{R^2-ab}\implies x=R-\sqrt{R^2-ab}.  	\]

    \[\quad\]

    \[\quad\]

Figura 36: applicazione del teorema delle Corde alle corde \overline{AB} e \overline{P_0F_0}.

    \[\quad\]

    \[\quad\]

quindi la risposta corretta è la 1.


 

Esercizio 89  (\bigstar\bigstar\bigstar\largewhitestar\largewhitestar). Sia V_1 il solido (corona sferica) delimitato da due superfici sferiche concentriche di raggi rispettivamente 2 e 5. Sia V_2 il cilindro circolare retto con raggio di base 4 e altezza 4. Detto x il rapporto fra il volume di V_1 e il volume di V_2, si ha che:

    \[\quad\]

  1. 3 < x < 4;
  2.  

  3. 4 < x < 5;
  4.  

  5. 2 < x < 3;
  6.  

  7. 5 < x < 6;
  8.  

  9. x < 2.

Svolgimento.

Risposta 3.

Il volume della corona sferica è dato dalla differenza tra il volume della sfera maggiore (r = 5) e quello della sfera minore (r = 2):

    \[ 	V_1 = \frac{4}{3} \pi (5^3 - 2^3) = \frac{4}{3} \pi (125 - 8) = \frac{4}{3} \pi 117 =  	156 \pi. 	\]

Il volume del cilindro con raggio r = 4 e altezza h = 4 è:

    \[ 	V_2 = \pi r^2 h = \pi \cdot 4^2 \cdot 4 = 64 \pi. 	\]

Il rapporto tra V_1 e V_2 è:

    \[ 	\frac{V_1}{V_2} = \frac{156 \pi}{64 \pi} = \frac{156}{64} = \frac{39}{16} \approx 2.43. 	\]

Quindi 2< x < 3 e la risposta corretta è la 3.


 

Esercizio 90  (\bigstar\bigstar\largewhitestar\largewhitestar\largewhitestar). Quale dei seguenti numeri ha logaritmo in base 10 strettamente compreso tra 5 e 6?

    \[\quad\]

  1. 10^4;
  2.  

  3. 10^5 - 1;
  4.  

  5. 18768766;
  6.  

  7. -10^5;
  8.  

  9. nessuna delle precedenti.

Svolgimento.

Risposta 5.

Ricordiamo che la funzione logaritmo è strettamente crescente, quindi se 0<x<y, allora \log_{10}(x)<\log_{10}(y).

    \[\quad\]

  • \log_{10}(10^4) = 4, che è minore di 5.
  •  

  • \log_{10}(10^5 - 1) <\log((10^5)=5.
  •  

  • \log_{10}(18768766)>\log_{10}(10^6)=6.
  •  

  • il logaritmo di un numero negativo non ha senso, quindi \log_{10}(-10^5) non si può determinare.

Nessuno dei casi proposti è corretto, quindi è vera la risposta 5.


 

Esercizio 91  (\bigstar\largewhitestar\largewhitestar\largewhitestar\largewhitestar). La disequazione \sqrt{3x - 2} < 2x - 1:

    \[\quad\]

  1. ha infinite soluzioni positive;
  2.  

  3. ha infinite soluzioni negative;
  4.  

  5. non ha soluzioni;
  6.  

  7. ha infinite soluzioni positive e infinite soluzioni negative;
  8.  

  9. ha un numero finito di soluzioni positive.

Svolgimento.

Risposta 1.

Per risolvere \sqrt{3x - 2} < 2x - 1, seguiamo questi passaggi:

    \[\quad\]

  1. La radice quadrata impone la condizione 3x - 2 \geq 0, quindi x \geq \dfrac{2}{3}, il che esclude 2 e 4.
  2.  

  3. Risolviamo la disequazione \sqrt{3x - 2} < 2x - 1 elevando al quadrato (che è possibile fare perché x\geq \dfrac{2}{3}\geq \dfrac{1}{2} e quindi 2x-1\geq 0)

        \[ 		3x - 2 < (2x - 1)^2 \quad \iff \quad 3x - 2 < 4x^2 - 4x + 1 		\]

    Portando tutto a destra:

        \[ 		0 < 4x^2 - 7x + 3 		\]

    Le soluzioni di questa disequazione di secondo grado sono x<\dfrac{3}{4} \vee x>1.

Intersecando l’insieme delle soluzioni con l’insieme dei valori x per cui x\geq\dfrac{2}{3}, otteniamo che la disequazione è valida per \dfrac{2}{3} \leq x < \dfrac{3}{4} oppure per x>1.

Quindi la risposta corretta è la 1.


 

Esercizio 92  (\bigstar\bigstar\largewhitestar\largewhitestar\largewhitestar). Data una retta r ortogonale a un piano \pi, quanti piani perpendicolari a \pi e contenenti r si possono condurre?

    \[\quad\]

  1. Uno;
  2.  

  3. Due;
  4.  

  5. Infiniti;
  6.  

  7. Nessuno;
  8.  

  9. Quattro.

Svolgimento.

Risposta 3.

Per una retta r ortogonale a un piano \pi è possibile condurre infiniti piani perpendicolari a \pi contenenti r. Ogni piano perpendicolare a \pi passante per la retta r può essere ruotato attorno a questa retta, generando un fascio di piani perpendicolari a \pi. Pertanto, la risposta corretta è la 3.


 

Esercizio 93  (\bigstar\bigstar\bigstar\largewhitestar\largewhitestar). Dire per quali numeri interi positivi \alpha e \beta esiste \gamma non multiplo di \beta tale che \alpha \gamma è multiplo di \beta.

    \[\quad\]

  1. \alpha e \beta pari;
  2.  

  3. \alpha e \beta dispari;
  4.  

  5. \alpha e \beta non primi tra loro;
  6.  

  7. per nessuna coppia di interi positivi;
  8.  

  9. \beta primo e \alpha dispari.

Svolgimento.

Risposta 3.

L’affermazione “esiste \gamma non multiplo di \beta tale che \alpha \gamma è multiplo di \beta” implica che \gamma non debba essere divisibile per almeno un fattore di \beta, ma che il prodotto \alpha \gamma lo sia. Ciò implica che \alpha deve avere almeno un fattore comune con \beta (quello che non ha con \gamma). Quindi \alpha e \beta non sono primi tra loro.

La risposta corretta è la 3.


 

Esercizio 94  (\bigstar\bigstar\bigstar\largewhitestar\largewhitestar). Dire quanti sono nel piano cartesiano (x, y) i punti le cui coordinate x e y soddisfano tutte e tre le condizioni seguenti:

    \[ 		x^2 + y^2 = 1, \qquad xy \geq 0, \qquad  x + y = 1. 		\]

    \[\quad\]

  1. Nessuno;
  2.  

  3. Uno;
  4.  

  5. Due;
  6.  

  7. Quattro;
  8.  

  9. Tre;

Svolgimento.

Risposta 3.

Consideriamo ciascuna delle tre condizioni:

    \[\quad\]

  1. La condizione xy \geq 0 implica che x e y devono avere lo stesso segno o almeno uno dei due deve essere 0. Quindi siamo nel primo e terzo quadrante compreso gli assi.
  2.  

  3. L’equazione x^2 + y^2 = 1 rappresenta una circonferenza di raggio 1 e centro nell’origine.
  4.  

  5. L’equazione x + y = 1 rappresenta una retta parallela alla bisettrice del secondo e quarto quadrante e passante per (0,1).

Risolvendo il sistema:

    \[ 	\begin{cases} 		x + y = 1 \\ 		x^2 + y^2 = 1 	\end{cases}. 	\]

Otteniamo i punti (0, 1) e (1, 0). Entrambi soddisfano xy \geq 0.

Pertanto, ci sono due punti che soddisfano tutte e tre le condizioni.

    \[\quad\]

    \[\quad\]

Figura 37: i tre insiemi descritti dalle tre equazioni e la loro intersezione.

    \[\quad\]

    \[\quad\]


 

Esercizio 95  (\bigstar\largewhitestar\largewhitestar\largewhitestar\largewhitestar). La nona parte di \left( \dfrac{1}{3} \right)^{50} è uguale a:

    \[\quad\]

  1. \left( \dfrac{1}{9} \right)^{25}
  2.  

  3. \left( \dfrac{1}{3} \right)^{51}
  4.  

  5. \left( \dfrac{1}{9} \right)^{50}
  6.  

  7. \left( \dfrac{1}{3} \right)^{52}
  8.  

  9. \left( \dfrac{1}{3} \right)^{48}

Svolgimento.

Risposta 4.

La nona parte di \left( \dfrac{1}{3} \right)^{50} equivale a moltiplicare \left( \dfrac{1}{3} \right)^{50} per \dfrac{1}{9}, che può essere riscritto come:

    \[ 	\frac{1}{9} \cdot \left( \frac{1}{3} \right)^{50} = \left( \frac{1}{3} \right)^2 \cdot \left( \frac{1}{3} \right)^{50} = \left( \frac{1}{3} \right)^{52} 	\]

Pertanto, la risposta corretta è la 4.


 

Esercizio 96  (\bigstar\bigstar\largewhitestar\largewhitestar\largewhitestar). La soluzione dell’equazione \log_{\dfrac{1}{49}} x = \frac{1}{4} è:

    \[\quad\]

  1. x = \dfrac{1}{7};
  2.  

  3. \dfrac{\sqrt{7}}{7};
  4.  

  5. x = 49;
  6.  

  7. x = -\dfrac{1}{7};
  8.  

  9. x=\dfrac{1}{49}.

Svolgimento.

Risposta 2.

Partiamo dall’equazione data:

    \[ 	\log_{\frac{1}{49}} x = \frac{1}{4} 	\]

Riscriviamo l’equazione in forma esponenziale utilizzando la definizione di logaritmo:

    \[ 	x = \left( \frac{1}{49} \right)^{\frac{1}{4}}. 	\]

Poiché \dfrac{1}{49} = 7^{-2}, possiamo scrivere:

    \[ 	x = \left( 7^{-2} \right)^{\frac{1}{4}} = 7^{-\frac{1}{2}} = \frac{\sqrt{7}}{7}.  	\]

Pertanto, la risposta corretta è la 2.


 

Esercizio 97  (\bigstar\bigstar\bigstar\largewhitestar\largewhitestar). Tre poligoni regolari di uguale perimetro (un quadrato, un ettagono e un dodecagono) sono inscritti in tre distinti cerchi. Quale dei tre cerchi ha la superficie maggiore?

    \[\quad\]

  1. Il cerchio con semiperimetro uguale al perimetro del poligono;
  2.  

  3. Il cerchio circoscritto al quadrato;
  4.  

  5. Il cerchio circoscritto al settagono;
  6.  

  7. Il cerchio circoscritto al dodecagono;
  8.  

  9. I tre cerchi hanno superfici identiche.

Svolgimento.

Risposta 2.

Dato che i poligoni hanno tutti lo stesso perimetro, il raggio del cerchio circoscritto diminuisce con il numero di lati del poligono. Più lati ha il poligono, più il raggio del cerchio circoscritto è piccolo. In questo caso, il quadrato ha il numero minore di lati, quindi avrà il raggio circoscritto maggiore. Pertanto, il cerchio con la superficie maggiore è quello circoscritto al quadrato.

Per giustificare meglio, calcoliamo il perimetro p di un poligono regolare di n lati inscritto ad un cerchio di raggio R. Se dividiamo a spicchi il poligono regolare, osserviamo che l’angolo al centro di ciascun spicchio è 360 gradi diviso il numero n dei lati, quindi per il Teorema della Corda un lato del poligono misura 2R\sin\left(\dfrac{\pi}{n}\right) e perciò il perimetro p risulterà uguale a

    \[ p=n\cdot 2R\sin\left(\frac{\pi}{n}\right). \]

Presi in considerazione due poligoni regolari inscritti in due cerchi di raggio R_1 ed R_2 ed aventi lo stesso perimetro p, allora se il primo poligono ha più lati del secondo, ovvero n_1>n_2, si ha

    \[ n_1\cdot 2R_1\sin\left(\frac{\pi}{n_1}\right)=p=n_2\cdot 2R_2\sin\left(\frac{\pi}{n_2}\right) \implies R_1=\frac{n_2}{n_1}\cdot \frac{\sin\left(\dfrac{\pi}{n_2}\right)}{\sin\left(\dfrac{\pi}{n_1}\right)}R_2\leq R_2. \]

L’ultima diseguaglianza segue da un risultato ben noto già dal liceo, ovvero che la funzione f(x)=\dfrac{\sin(x)}{x} è descrescente per x\in \left(0, \pi\right]. Nel nostro caso, il numero n di lati di un poligono regolare è almeno 3, quindi \dfrac{\pi}{n}\in \left(0, \dfrac{\pi}{2}\right] e perciò se n_1> n_2, ovvero \dfrac{\pi}{n_2}\geq \dfrac{\pi}{n_1}, allora

    \[ \frac{\sin\left(\dfrac{\pi}{n_2}\right)}{\dfrac{\pi}{n_2}}\leq \dfrac{\sin\left(\dfrac{\pi}{n_1}\right)}{\dfrac{\pi}{n_1}} \implies  \dfrac{n_2\cdot \sin\left(\dfrac{\pi}{n_2}\right)}{n_1\cdot \sin\left(\dfrac{\pi}{n_1}\right)}\leq 1.  \]

Possiamo concludere quindi che il cerchio circoscritto nel poligono con meno lati è anche quello che ha raggio maggiore, e quindi superficie maggiore.

    \[\quad\]

    \[\quad\]

Figura 38: i tre poligoni regolari di uguale perimetro circoscritti alle loro circonferenze.

    \[\quad\]

    \[\quad\]

La risposta corretta è la 2.


 

Esercizio 98  (\bigstar\bigstar\largewhitestar\largewhitestar\largewhitestar). È vero che \dfrac{\alpha}{\beta} + \dfrac{\beta}{\alpha} \geq 2, comunque scelti \alpha e \beta interi non nulli?

    \[\quad\]

  1. Mai;
  2.  

  3. Sì, con \alpha > \beta > 2.
  4.  

  5. Sì, per qualsiasi valore di \alpha e di \beta con lo stesso segno;
  6.  

  7. Sì, con \alpha > 1 e \beta qualsiasi;
  8.  

  9. Sì, con \alpha > 1 e \beta> 1;

Svolgimento.

Risposta 3.

Supponiamo che \alpha e \beta abbiano lo stesso segno, quindi che il rapporto fra loro sia positivo.

Riscriviamo l’espressione:

    \[ 	\frac{\alpha}{\beta} + \frac{\beta}{\alpha} \geq 2 \iff  \frac{\alpha^2 + \beta^2-2\alpha\beta}{\alpha\beta}\geq 0 \iff \frac{(\alpha-\beta)^2}{\alpha\beta}\geq 0 \iff \alpha\beta> 0, 	\]

dato che (\alpha-\beta)^2 è sempre maggiore o uguale a zero.

La risposta corretta è la 3.


 

Esercizio 99  (\bigstar\bigstar\bigstar\largewhitestar\largewhitestar). Lanciando due dadi regolari, quale tra i sottoelencati risultati ha la più alta probabilità di verificarsi?

    \[\quad\]

  1. 12;
  2.  

  3. 4;
  4.  

  5. 9;
  6.  

  7. 8;
  8.  

  9. 11.

Svolgimento.

Risposta 4.

Quando si lanciano due dadi, le somme possibili vanno da 2 a 12, e ognuna ha una probabilità diversa a seconda di quante combinazioni producono quella somma. Inoltre, più combinazione di quella somma ci sono, più è probabile che quella somma esca. Ecco le combinazioni per ogni somma:

    \[\quad\]

  • Somma 2: (1, 1), ovvero una combinazione;
  •  

  • Somma 3: (1, 2), (2, 1), ovvero 2 combinazioni;
  •  

  • Somma 4: (1, 3), (2, 2), (3, 1) ovvero 3 combinazioni;
  •  

  • Somma 5: (1, 4), (2, 3), (3, 2), (4, 1), ovvero 4 combinazioni;
  •  

  • Somma 6: (1, 5), (2, 4), (3, 3), (4, 2), (5, 1), ovvero 5 combinazioni;
  •  

  • Somma 7: (1, 6), (2, 5), (3, 4), (4, 3), (5, 2), (6, 1), ovvero 6 combinazioni;
  •  

  • Somma 8: (2, 6), (3, 5), (4, 4), (5, 3), (6, 2), ovvero 5 combinazioni;
  •  

  • Somma 9: (3, 6), (4, 5), (5, 4), (6, 3), ovvero 4 combinazioni;
  •  

  • Somma 10: (4, 6), (5, 5), (6, 4), ovvero 3 combinazioni;
  •  

  • Somma 11: (5, 6), (6, 5), ovvero 2 combinazioni;
  •  

  • Somma 12: (6, 6), ovvero 1 combinazione.

La somma con la più alta probabilità di verificarsi tra le possibili opzioni è la somma 8, che ha 5 combinazioni.

La risposta corretta è la 4.


 

Esercizio 100  (\bigstar\largewhitestar\largewhitestar\largewhitestar\largewhitestar). In un triangolo rettangolo, il quadrato costruito su un cateto è equivalente al rettangolo che ha per lati:

    \[\quad\]

  1. Il cateto stesso e la differenza fra l’ipotenusa e l’altro cateto;
  2.  

  3. L’ipotenusa e la proiezione dell’altro cateto sull’ipotenusa;
  4.  

  5. la differenza fra l’ipotenusa e l’altro cateto, e la somma dell’ipotenusa con l’altro cateto;
  6.  

  7. Il cateto stesso e la differenza delle proiezioni dei cateti sull’ipotenusa;
  8.  

  9. Il cateto stesso e la proiezione dell’altro cateto sull’ipotenusa.

Svolgimento.

Risposta 3.

Consideriamo un triangolo di cateti a e b e ipotenusa c. Per il teorema di Pitagora

    \[ 	c^2=a^2+b^2 \implies b^2=c^2-a^2=(c-a)(c+a). 	\]

La risposta corretta è la 3.

 

Esercizio 101  (\bigstar\bigstar\largewhitestar\largewhitestar\largewhitestar). Perché una retta sia ortogonale a un piano è sufficiente che:

    \[\quad\]

  1. Sia ortogonale a una retta del piano;
  2.  

  3. Sia parallela a una retta del piano;
  4.  

  5. I suoi punti siano equidistanti da tutti i punti del piano;
  6.  

  7. I suoi punti non siano equidistanti dai punti del piano;
  8.  

  9. Sia ortogonale a due rette incidenti del piano.

Svolgimento.

Risposta 5.

Una retta è ortogonale a un piano se è ortogonale a due rette incidenti del piano. Tale retta si può costruire come intersezione tra i due piani conententi una delle due rette rispettivemente ed ortogonali al piano dato.

Pertanto, la risposta corretta è la 5.

 

Esercizio 102  (\bigstar\largewhitestar\largewhitestar\largewhitestar\largewhitestar). Elevando alla quarta i due membri di un’equazione si ottiene un’equazione che:

    \[\quad\]

  1. Ha le stesse soluzioni di quella iniziale;
  2.  

  3. Può avere meno soluzioni di quella iniziale;
  4.  

  5. Può avere più soluzioni di quella iniziale;
  6.  

  7. Non ha alcuna relazione con l’equazione iniziale;
  8.  

  9. Ha infinite soluzioni.

Svolgimento.

Risposta 3.

Elevando alla quarta entrambi i membri di un’equazione, questa può avere soluzioni aggiuntive che non soddisfano l’equazione originale. Questo accade perché l’elevazione alla quarta di un numero positivo e quello di un numero negativo producono lo stesso valore. Ad esempio l’equazione x=1 ha una sola solzione, mentre x^4=1^4=1 ha due soluzioni reali, ovvero x=\pm 1.

Pertanto, l’equazione alla quarta può avere più soluzioni rispetto a quella iniziale.

La risposta corretta è la 3.

 

Esercizio 103  (\bigstar\largewhitestar\largewhitestar\largewhitestar\largewhitestar). Di un triangolo si conoscono i tre lati a, b, c e gli angoli \alpha, \beta, \gamma, ordinatamente opposti ai lati (con \alpha > \beta > \gamma). Quale delle seguenti affermazioni è vera?

    \[\quad\]

  1. a + c= b;
  2.  

  3. a+b = c;
  4.  

  5. c+b=a;
  6.  

  7. a>b>c;
  8.  

  9. nessuna delle precedenti.

Svolgimento.

Risposta 4.

Poiché gli angoli del triangolo sono ordinati in modo decrescente \alpha >\beta > \gamma, anche i lati opposti sono ordinati nello stesso modo, quindi a > b> c. Ciò vale per un teorema ben noto di Geometria per cui in un triangolo ad angolo maggiore si oppone lato maggiore.

La risposta corretta è la 4.

 

Esercizio 104  (\bigstar\bigstar\largewhitestar\largewhitestar\largewhitestar). Per quanti numeri interi positivi n, la potenza 13^n è un numero con meno di 13 cifre?

    \[\quad\]

  1. Uno;
  2.  

  3. Due;
  4.  

  5. Tre;
  6.  

  7. Più di tre;
  8.  

  9. Più di tre, ma comunque un numero finito.

Svolgimento.

Risposta 5.

Il numero di cifre di un numero x si calcola con la formula:

    \[ \text{Numero di cifre} = \lfloor \log_{10}(x) \rfloor + 1, \]

dove il simbolo \lfloor \cdot \rfloor indica la parte intera inferiore di un numero. Per parte intera inferiore di un numero x si intende il più grande numero intero più piccolo di x.

Quindi, dobbiamo risolvere la disequazione:

    \[ \lfloor \log_{10}(13^n) \rfloor + 1 < 13 \iff \lfloor \log_{10}(13^n) \rfloor <12 \iff \log_{10}(13^n)<12.  \]

Poiché \log_{10}(13^n) = n \log_{10}(13), la disequazione diventa:

    \[ n<n \log_{10}(13)  < 12. \]

Quindi n non può superare 11, ma sicuramente può superare 3.

Effetivamente, con l’uso di una calcolatrice si può calcolare il primo numero n per cui 13^n ha più di 13 cifre, e questo è 11:

    \[ n \log_{10}(13) < 12 \iff  n < \frac{12}{\log_{10}(13)} \approx \frac{12}{1.11} \approx 10.77. \]

La risposta corretta è la 5.

 

Esercizio 105  (\bigstar\bigstar\largewhitestar\largewhitestar\largewhitestar). Se una grandezza x è direttamente proporzionale al cubo di una grandezza y, e y è inversamente proporzionale al quadrato di una grandezza z, allora:

    \[\quad\]

  1. x è inversamente proporzionale alla sesta di z;
  2.  

  3. x è direttamente proporzionale al cubo di z;
  4.  

  5. x è direttamente proporzionale al quadrato di z;
  6.  

  7. x è inversamente proporzionale a z;
  8.  

  9. La relazione tra x e y è diversa da quelle indicate nelle altre risposte.

Svolgimento.

Risposta 1.

Ci sono due relazioni:

    \[\quad\]

  1. x è direttamente proporzionale al cubo di y, quindi:

        \[ 		x = k_1 y^3 		\]

  2.  

  3. y è inversamente proporzionale al quadrato di z, quindi:

        \[ 		yz^2=k_2 \implies z^2= \frac{k_2}{y} \implies z^6=\frac{k_2^3}{y^3}. 		\]

Allora

    \[ 	xz^6 = k_1 y^3\frac{k_2^3}{y^3}  = k_1k_2^3. 	\]

Dunque x è inversamente proporzionale alla sesta di z.

La risposta corretta è la 1.

 

Esercizio 106  (\bigstar\largewhitestar\largewhitestar\largewhitestar\largewhitestar). L’equazione (\log_3 x)^2 - \log_3 x = 1 è verificata da:

    \[\quad\]

  1. Da nessun valore reale;
  2.  

  3. Da due valori reali;
  4.  

  5. Da un solo valore reale;
  6.  

  7. Infiniti valori reali positivi;
  8.  

  9. Nessuna delle risposte precedenti.

Svolgimento.

Risposta 2.

L’equazione è:

    \[ 	(\log_3 x)^2 - \log_3 x = 1. 	\]

Poniamo y = \log_3 x, così che l’equazione diventi:

    \[ 	y^2 - y - 1 = 0. 	\]

Risolvendo l’equazione di secondo grado:

    \[ 	y = \frac{-(-1) \pm \sqrt{(-1)^2 - 4 \cdot 1 \cdot (-1)}}{2 \cdot 1} = \frac{1 \pm \sqrt{5}}{2} \implies y_1 = \frac{1 + \sqrt{5}}{2}, \quad y_2 = \frac{1 - \sqrt{5}}{2}. 	\]

Ora, poiché y = \log_3 x, otteniamo:

    \[ 	\log_3(x)=\frac{1 \pm  \sqrt{5}}{2}  \implies x=3^{\frac{1 \pm \sqrt{5}}{2}}. 	\]

Pertanto, l’equazione ha due soluzione reali.

La risposta corretta è 2.

 

Esercizio 107  (\bigstar\bigstar\bigstar\largewhitestar\largewhitestar). Tre amici Alice, Bob e Carlo (per brevità A, B e C) giocano a testa o croce con una moneta. Ciascuno tira una sola volta la moneta: prima tira A, poi tira B e per ultimo C; vince chi fa croce per primo. Dette rispettivamente P_A, P_B, P_C le probabilità di vittoria di A, B e C, risulta:

    \[\quad\]

  1. P_A = P_B = P_C = 50\%;
  2.  

  3. P_A = 2P_B = 3P_C;
  4.  

  5. P_A = P_B = P_C = 33.3\% (circa);
  6.  

  7. P_A + P_B + P_C = 100\%;
  8.  

  9. P_A = 2P_B = 4P_C.

Svolgimento.

Risposta 5.

La probabilità che A vinca è che faccia croce, ovvero:

    \[ 	P_A = \frac{1}{2}. 	\]

La probabilità che B vinca è che A faccia testa per la probabilità che B faccia croce (ovvero che su due tiri esca prima testa e poi croce). Dato che i due lanci sono indipendenti, allora:

    \[ 	P_B = P_{primo  \ tiro \  testa}\cdot P_{secondo \ tiro \ croce}=\frac{1}{2} \cdot \frac{1}{2} = \frac{1}{4}. 	\]

La probabilità che C vinca è che A faccia testa, B faccia testa e che C faccia croce (ovvero che su tre tiri esca in sequenza testa, testa croce). Di nuovo, essendo i tre lanci indipendenti, allora

    \[ 	P_C = P_{primo  \ tiro \  testa}\cdot P_{secondo \ tiro \ testa}\cdot P_{terzo \ tiro \ croce}= \frac{1}{2} \cdot \frac{1}{2} \cdot \frac{1}{2} = \frac{1}{8}.  	\]

La relazione fra le probabilità ottenute è:

    \[ 	P_A=2P_B=4P_C.  	\]

Pertanto, la risposta corretta è la 5.

 

Esercizio 108  (\bigstar\bigstar\bigstar\largewhitestar\largewhitestar). In un quadrato Q si consideri il quadrato inscritto che ha come vertici i punti medi dei lati di Q. A partire da questo secondo quadrato se ne costruisca un terzo con lo stesso procedimento, e così via. Quante volte al massimo si può ripetere il procedimento descritto per ottenere alla fine un quadrato il cui perimetro non sia più piccolo della duecentesima parte di quello di Q?

    \[\quad\]

  1. 15;
  2.  

  3. Più di 1000;
  4.  

  5. 25;
  6.  

  7. 1000;
  8.  

  9. 100.

Svolgimento.

Risposta 1.

Quando si inscrive un quadrato dentro un altro quadrato con i punti medi dei lati, il lato l_1 del nuovo quadrato è dal teorema di Pitagora uguale a l_1=\sqrt{\dfrac{l^2}{4}+\dfrac{l^2}{4}}=\dfrac{\sqrt{2}}{2}l, dove l è il lato del quadrato iniziale. Dunque il perimetro p_1 del nuovo quadrato sarà

    \[ 	p_1=4\cdot \frac{\sqrt{2}}{2}l=\frac{\sqrt{2}}{2}p, 	\]

dove p è il perimetro del quadrato iniziale. Ripetendo lo stesso procedimento per l_2, otteniamo p_2=\dfrac{\sqrt{2}}{2}p_1=\dfrac{1}{2}p. Iterando ad ogni passo, otteniamo che l’n-esimo quadrato avrà perimetro uguale a

    \[ 	p_n=\sqrt{2^{-n}}\cdot p.  	\]

Dobbiamo quindi risolvere la disequazione

    \[ 	p_n\geq \frac{p}{200} \implies \sqrt{2^{-n}}\cdot p\geq \frac{p}{200}  \implies \sqrt{2^{n}}\leq 200 \implies 2^{n}\leq 2^6\cdot 5^4. 	\]

Quindi

    \[ 	2^{n}\leq 2^6\cdot 5^4 \implies n\leq \log_2(2^6\cdot 5^4)=\log_2(2^6)+4\log_2(5)< 6+9.3=15.3. 	\]

Dunque si può iterare il procedimento fino ad un massimo di 15 volte.

    \[\quad\]

    \[\quad\]

Figura 39: un esempio di sequenza di 3 quadrati.

    \[\quad\]

    \[\quad\]

La risposta corretta è la 1.


 

Esercizio 109  (\bigstar\bigstar\bigstar\largewhitestar\largewhitestar). Dati 4 mazzi di carte, ciascuno dei quali è composto da 40 carte contenenti 4 assi di seme diverso, calcolare la probabilità, estraendo una carta da ogni mazzo, di estrarre da ciascuno di essi l’asso di quadri o l’asso di fiori.

    \[\quad\]

  1. \dfrac{1}{40};
  2.  

  3. \dfrac{1}{5};
  4.  

  5. \dfrac{1}{10};
  6.  

  7. \dfrac{1}{160000};
  8.  

  9. \dfrac{1}{8000}.

Svolgimento.

Risposta 4.

Ogni mazzo contiene 40 carte e 4 assi (uno per seme: cuori, quadri, fiori e picche). La probabilità di estrarre l’asso di quadri o l’asso di fiori da un mazzo è:

    \[ 	P(\text{asso di picche o cuori}) = \frac{1}{40} + \frac{1}{40} = \frac{2}{40} = \frac{1}{20}. 	\]

La probabilità di estrarre da ciascuno dei 4 mazzi un asso di quadri o un asso di fiori è:

    \[ 	P_{\text{tot}} = \left(\frac{1}{20}\right) \cdot \left(\frac{1}{20}\right) \cdot \left(\frac{1}{20}\right) \cdot \left(\frac{1}{20}\right) = \frac{1}{20^4} = \frac{1}{160000}. 	\]

La risposta corretta è la 4.


 

Esercizio 110  (\bigstar\bigstar\largewhitestar\largewhitestar\largewhitestar). Nel gioco della roulette i numeri vanno da 0 a 36. Qual è la probabilità che escano il 16 e il 18 in due lanci?

    \[\quad\]

  1. \dfrac{1}{37 \cdot 36};
  2.  

  3. \dfrac{2}{37 \cdot 37};
  4.  

  5. \dfrac{1}{18 \cdot 36};
  6.  

  7. \dfrac{1}{37} + \dfrac{1}{37};
  8.  

  9. \dfrac{1}{17 \cdot 17}.

Svolgimento.

Risposta 2.

La probabilità che esca il numero 16 o il 18 in una singola estrazione è \dfrac{1}{37}. La probabilità che escano in due lanci i numeri desiderati è la probabilità che esca prima il 16 e poi il 18 oppure prima il 18 e poi il 16. Quindi:

    \[ 	P(\text{16 \text{e}  18}) = P((16,18))+P((18,16))= 2 \cdot \frac{1}{37}\cdot \frac{1}{37} = \frac{2}{37^2} =\frac{2}{1369}. 	\]

La risposta corretta è la 2.


 

Esercizio 111  (\bigstar\bigstar\largewhitestar\largewhitestar\largewhitestar). Nel piano Oxy le equazioni y = -152 e y = 3x^2 rappresentano:

    \[\quad\]

  1. due rette parallele;
  2.  

  3. una retta e un’iperbole che non si incontrano;
  4.  

  5. una retta e un’iperbole che si incontrano in due punti;
  6.  

  7. una retta e una parabola che si incontrano in due punti;
  8.  

  9. una retta e una parabola che non si incontrano.

Svolgimento.

Risposta 5.

Le equazioni date sono:

    \[ 	y = -152\quad \text{(equazione di una retta orizzontale)} \qquad  \makebox{e} \qquad y =3 x^2 \quad \text{(equazione di una parabola)}. 	\]

Uguagliando le due equazioni:

    \[ 	x^2 = -\frac{152}{3},  	\]

che non ha soluzioni reali, poiché il quadrato di un numero reale non può essere negativo.

La risposta corretta è 5.


 

Esercizio 112  (\bigstar\largewhitestar\largewhitestar\largewhitestar\largewhitestar). Quale dei seguenti insiemi numerici è formato da un solo elemento?

    \[\quad\]

  1. \{x \in\mathbb R \colon   x^2 =-x\};
  2.  

  3. \{x \in\mathbb R \colon  x \leq  13\};
  4.  

  5. \{x \in\mathbb R \colon  x^2 = -2x - 1\};
  6.  

  7. \{x \in\mathbb R \colon x^2 + 1 = 0\};
  8.  

  9. \{x \in\mathbb R \colon x^2 - 12 = 0\}.

Svolgimento.

Risposta 3.

  1. x^2 = -x

    Questa equazione ha due soluzioni: x = 0 e x = -1. L’insieme è composto di due elementi.

  2.  

  3. x \leq 13

    Questo insieme contiene infiniti numeri reali minori o uguali a 13, quindi non è un insieme con un solo elemento.

  4.  

  5. x^2 = -2x -1

    Risolvendo l’equazione (x -1)^2 = 0, troviamo l’unica soluzione x = -1. L’insieme contiene un solo elemento.

  6.  

  7. x^2 + 1 = 0

    Questa equazione non ha soluzioni reali, quindi l’insieme è vuoto.

  8.  

  9. x^2 - 12 = 0

    Le soluzioni sono x = \sqrt{12} e x = -\sqrt{12}, quindi l’insieme contiene due elementi.

La risposta corretta è la 3.


 

Esercizio 113  (\bigstar\largewhitestar\largewhitestar\largewhitestar\largewhitestar). Date due funzioni reali di variabile reale f, g \colon \mathbb R\to \mathbb R, l’identità f = g vuol dire che:

    \[\quad\]

  1. esiste x\in \mathbb R tale che f(x)=g(x);
  2.  

  3. per ogni x\in\mathbb R si ha f(x)=g(x);
  4.  

  5. esistono infiniti valori x\in\mathbb R per cui f(x)=g(x);
  6.  

  7. per rispondere occorre conoscere le espressioni di f e di g;
  8.  

  9. nessuna delle precedenti è corretta.

Svolgimento.

Risposta 2.

L’identità data significa che per ogni valore x\in \mathbb R si ha che f(x)=g(x).

La risposta corretta è la 2.


 

Esercizio 114  (\bigstar\largewhitestar\largewhitestar\largewhitestar\largewhitestar). La media aritmetica tra \left( \dfrac{1}{3} \right)^{-3} e \left( \dfrac{1}{3} \right)^3 è:

    \[\quad\]

  1. uguale a 0;
  2.  

  3. minore di 0;
  4.  

  5. uguale a \dfrac{365}{27};
  6.  

  7. uguale a \dfrac{730}{27};
  8.  

  9. uguale a \dfrac{7}{4}.

Svolgimento.

Risposta 3.

Calcoliamo \left( \dfrac{1}{3} \right)^{-3} e \left( \dfrac{1}{3} \right)^3:

    \[ 	\left(\frac{1}{3}\right)^{-3}=\left( \frac{3}{1} \right)^3 = 27 \qquad \makebox{e} \qquad \left( \frac{1}{3} \right)^3= \frac{1}{27}. 	\]

La media aritmetica è:

    \[ 	\frac{27 + \dfrac{1}{27}}{2} = \dfrac{\dfrac{27^2+1}{27}}{2} = \frac{365}{27}.  	\]

La risposta corretta è C).


 

Esercizio 115  (\bigstar\largewhitestar\largewhitestar\largewhitestar\largewhitestar). Per quali valori di x risulta x^3 > 216?

    \[\quad\]

  1. x > 6;
  2.  

  3. x < -6 \, , \, x > 6;
  4.  

  5. -6 < x < 6;
  6.  

  7. x > -6;
  8.  

  9. x < -6.

Svolgimento.

Risposta 1.

L’uguaglianza x^3 = 216 ha soluzione x=6. Quindi la disequazione x^3 > 216 è verificata per tutti i valori di x maggiori di 6.

La risposta corretta è la 1.


 

Esercizio 116  (\bigstar\largewhitestar\largewhitestar\largewhitestar\largewhitestar). Supponiamo di avere due eventi A e B tali che P(A) = 0.3, P(A \cup B) = 0.7 e P(A\cap B)=0.1. Allora:

    \[\quad\]

  1. P(B) = 1;
  2.  

  3. P(B) = 0.4;
  4.  

  5. la probabilità di B non è calcolabile;
  6.  

  7. P(B) = 0.5;
  8.  

  9. P(B) = 0.3.

Svolgimento.

Risposta 4.

Possiamo usare la seguente formula ben nota in probabilità:

    \[ 	P(A \cup B) = P(A) + P(B)-P(A\cap B) \implies P(B)=P(A \cup B) +P(A \cap B) -P(A). 	\]

Sostituendo i valori dati:

    \[ 	P(B)=0.7+0.1-0.3=0.5 	\]

La risposta corretta è 4.


 

Esercizio 117  (\bigstar\largewhitestar\largewhitestar\largewhitestar\largewhitestar). Una curva di equazione x^2y^2 = k (k > 0) e una retta di equazione x + y = h hanno in comune:

    \[\quad\]

  1. un punto;
  2.  

  3. due punti;
  4.  

  5. nessun punto;
  6.  

  7. dipende dal segno di h;
  8.  

  9. nessuna delle precedenti.

Svolgimento.

Risposta 5.

La curva data è l’unione di due iperboli equilatere di coefficiente \sqrt{k} e -\sqrt{k} rispettivamente.

La retta data invece è parallela alla bisettrice del secondo e quarto quadrante e passa per il punto (0,h). Ci aspettiamo quindi che per ogni valore di h la retta interseca in due punti xy=-\sqrt{k}, mentre solo per alcuni valori di h (che dipenderanno da k) avremo che la retta interseca anche l’iperbole xy=\sqrt{k}.

Per determinare in modo esatto per quali valori, sostituiamo y = -x +h nell’equazione dell’iperbole e risolviamo l’equazione in x:

    \[ 	x(-x +h) = \sqrt{k} \implies -x^2+hx=\sqrt{k} \implies x^2-hx+\sqrt{k}=0. 	\]

Il numero dei punti di intersezione dipendono quindi dal segno del discriminante \Delta dell’equazione di secondo grado ottenuta:

    \[ 	\Delta = h^2 - 4\sqrt{k}\geq 0 \implies h\leq -2\sqrt[4]{k} \qquad \makebox{oppure} \qquad h\geq 2\sqrt[4]{k}.  	\]

Quindi il numero di punti di intersezione fra le due curve sarà:

    \[\quad\]

    2 se -2\sqrt[4]{k}<h< 2\sqrt[4]{k};

     

  • 3 se h=-2\sqrt[4]{k} oppure h=2\sqrt[4]{k};
  •  

  • 4 se h< -2\sqrt[4]{k} oppure h\geq 2\sqrt[4]{k}.

    \[\quad\]

    \[\quad\]

Figura 40: la retta e le due iperboli equilatere nel caso k=4 e h=3.

    \[\quad\]

    \[\quad\]

La risposta corretta è 5.


 

Esercizio 118  (\bigstar\largewhitestar\largewhitestar\largewhitestar\largewhitestar). Trovare l’area del triangolo compreso fra gli assi cartesiani e la retta di equazione:

    \[ 		y = 7 - \frac{x}{3} 		\]

    \[\quad\]

  1. \dfrac{7}{4};
  2.  

  3. \dfrac{3}{7};
  4.  

  5. \dfrac{134}{2};
  6.  

  7. \dfrac{147}{2};
  8.  

  9. Nessuna delle precedenti alternative è corretta.

Svolgimento.

Risposta 4.

Per trovare i punti di intersezione della retta data con gli assi, facciamo i seguenti calcoli:

    \[\quad\]

  • Intersezione con l’asse x (y = 0):

        \[ 		0 = 7 - \frac{x}{3} \quad \Rightarrow \quad x = 21.  		\]

    Il punto di intersezione è (21, 0).

  •  

  • Intersezione con l’asse y (x = 0):

        \[ 		y = 7 - \frac{0}{2} = 7.  		\]

    Il punto di intersezione è (0, 7).

L’area del triangolo è:

    \[ 	A= \frac{1}{2} \times \text{base} \times \text{altezza} = \frac{1}{2} \times 21\times 7 = \frac{147}{2}.  	\]

La risposta corretta è 4.


 

Esercizio 119  (\bigstar\largewhitestar\largewhitestar\largewhitestar\largewhitestar). Quanto vale l’espressione:

    \[ 		\dfrac{73^{56}}{73^{-21} - 73^{20}} \cdot (73^3)^{-2}? 		\]

    \[\quad\]

  1. \dfrac{73^{41}}{1-73^{ 					71}};
  2.  

  3. \dfrac{73^{71}}{1-73^{41}};
  4.  

  5. \dfrac{73^{21}}{1-73^{41}};
  6.  

  7. \dfrac{73^{41}}{1-73^{41}};
  8.  

  9. Nessuna delle precedenti alternative è corretta.

Svolgimento.

Risposta 2.

Riscriviamo l’espressione:

    \[ 	\frac{73^{56}}{73^{-21} - 73^{20}} \cdot (73^3)^{-2}=\frac{73^{56-6}}{\frac{1-73^{41}}{73^{21}}}= \frac{73^{50+21}}{1-73^{41}}=\frac{73^{71}}{1-73^{41}}. 	\]

La risposta corretta tra quelle elencate è la 2.


 

Esercizio 120  (\bigstar\largewhitestar\largewhitestar\largewhitestar\largewhitestar). La funzione f(x) = \frac{x - 1}{x^2+4x-5} è definita:

    \[\quad\]

  1. per ogni x reale;
  2.  

  3. per ogni x diverso da -5;
  4.  

  5. per ogni x diverso da 1 e da -5;
  6.  

  7. per x > 0;
  8.  

  9. per x < 0.

Svolgimento.

Risposta 3.

La funzione f(x) = \dfrac{x - 1}{x^2+4x-5} non è definita dove il denominatore si annulla. La funzione è definita quindi solo quando x^2+4x-5 \neq 0. Risolviamo l’equazione di secondo grado ottenendo:

    \[ 	x_1=1 \qquad \makebox{e} \qquad x_2=-5.  	\]

Pertanto f(x) è definita per ogni x diverso da 1 e -5. Si noti che la funzione f può essere riscritta come

    \[ 	f(x)=\frac{x-1}{(x-1)(x+5)},  	\]

che è tuttavia diversa dalla funzione g(x)=\dfrac{1}{x+5}. Infatti le funzioni f e g coincidono al di fuori di x=1 ed in particolare g è un’estensione di f in x=1 che, a differenza di f, è continua in x=1.

    \[\quad\]

    \[\quad\]

    \[\quad\]

    \[\quad\]

La risposta corretta è la 3.


 
 

Tutta la teoria di analisi matematica

Leggi...

  1. Teoria Insiemi
  2. Il metodo della diagonale di Cantor
  3. Logica elementare
  4. Densità dei numeri razionali nei numeri reali
  5. Insiemi Numerici \left(\mathbb{N},\, \mathbb{Z},\, \mathbb{Q}\right)
  6. Il principio di induzione
  7. Gli assiomi di Peano
  8. L’insieme dei numeri reali: costruzione e applicazioni
  9. Concetti Fondamentali della Retta Reale: Sintesi Teorica
  10. Costruzioni alternative di \mathbb{R}
  11. Binomio di Newton
  12. Spazi metrici, un’introduzione
  13. Disuguaglianza di Bernoulli
  14. Disuguaglianza triangolare
  15. Teoria sulle funzioni
  16. Funzioni elementari: algebriche, esponenziali e logaritmiche
  17. Funzioni elementari: trigonometriche e iperboliche
  18. Funzioni goniometriche: la guida essenziale
  19. Teorema di Bolzano-Weierstrass per le successioni
  20. Criterio del rapporto per le successioni
  21. Definizione e proprietà del numero di Nepero
  22. Limite di una successione monotona
  23. Successioni di Cauchy
  24. Il teorema ponte
  25. Teoria sui limiti
  26. Simboli di Landau
  27. Funzioni continue – Teoria
  28. Il teorema di Weierstrass
  29. Il teorema dei valori intermedi
  30. Il teorema della permanenza del segno
  31. Il teorema di Heine-Cantor
  32. Il teorema di esistenza degli zeri
  33. Il metodo di bisezione
  34. Teorema ponte versione per le funzioni continue
  35. Discontinuità di funzioni monotone
  36. Continuità della funzione inversa
  37. Teorema delle contrazioni o Teorema di punto fisso di Banach-Caccioppoli
  38. Teoria sulle derivate
  39. Calcolo delle derivate: la guida pratica
  40. Teoria sulle funzioni convesse
  41. Il teorema di Darboux
  42. I teoremi di de l’Hôpital
  43. Teorema di Fermat
  44. Teoremi di Rolle e Lagrange
  45. Il teorema di Cauchy
  46. Espansione di Taylor: teoria, esempi e applicazioni pratiche
  47. Polinomi di Taylor nei limiti: istruzioni per l’uso
  48. Integrali definiti e indefiniti
  49. Teorema fondamentale del calcolo integrale (approfondimento)
  50. Integrali ricorsivi
  51. Formule del trapezio, rettangolo e Cavalieri-Simpson
  52. Teoria sugli integrali impropri
  53. Funzioni integrali – Teoria
  54. Introduzione ai numeri complessi – Volume 1 (per un corso di ingegneria — versione semplificata)
  55. Introduzione ai numeri complessi – Volume 1 (per un corso di matematica o fisica)
  56. Serie numeriche: la guida completa
  57. Successioni di funzioni – Teoria
  58. Teoremi sulle successioni di funzioni
    1. 58a. Criterio di Cauchy per la convergenza uniforme
    2. 58b. Limite uniforme di funzioni continue
    3. 58c. Passaggio al limite sotto il segno di integrale
    4. 58d. Limite uniforme di funzioni derivabili
    5. 58e. Piccolo teorema del Dini
    6. 58f. Procedura diagonale e teorema di Ascoli-Arzela
  59. Serie di funzioni – Teoria
  60. Serie di potenze – Teoria
  61. Serie di Fourier – Teoria e applicazioni
  62. Integrali multipli — Parte 1 (teoria)
  63. Integrali multipli — Parte 2 (teoria e esercizi misti)
  64. Regola della Catena — Teoria ed esempi.
  65. Jacobiano associato al cambiamento di coordinate sferiche
  66. Guida ai Massimi e Minimi: Tecniche e Teoria nelle Funzioni Multivariabili
  67. Operatore di Laplace o Laplaciano
  68. Teoria equazioni differenziali
  69. Equazione di Eulero
  70. Teoria ed esercizi sulla funzione Gamma di Eulero
  71. Teoria ed esercizi sulla funzione Beta
  72. Approfondimento numeri complessi
  73. Diverse formulazioni dell’assioma di completezza
  74. Numeri di Delannoy centrali
  75. Esercizi avanzati analisi

 
 

Tutte le cartelle di Analisi Matematica

Leggi...

  1. Prerequisiti di Analisi
    1. Ripasso algebra biennio liceo
    2. Ripasso geometria analitica
    3. Ripasso goniometria e trigonometria
    4. Errori tipici da evitare
    5. Insiemi numerici N,Z,Q,R
    6. Funzioni elementari
    7. Logica elementare
    8. Insiemi
  2. Successioni
    1. Teoria sulle Successioni
    2. Estremo superiore e inferiore
    3. Limiti base
    4. Forme indeterminate
    5. Limiti notevoli
    6. Esercizi misti Successioni
    7. Successioni per ricorrenza
  3. Funzioni
    1. Teoria sulle funzioni
    2. Verifica del limite in funzioni
    3. Limite base in funzioni
    4. Forme indeterminate in funzioni
    5. Limiti notevoli in funzioni
    6. Calcolo asintoti
    7. Studio di funzione senza derivate
    8. Dominio di una funzione
    9. Esercizi misti Funzioni
    10. Esercizi misti sui Limiti
  4. Funzioni continue-lipschitziane-holderiane
    1. Teoria sulle Funzioni continue-lipschitziane-holderiane
    2. Continuità delle funzioni
    3. Continuità uniforme
    4. Teorema degli zeri
    5. Esercizi sul teorema di Weierstrass senza l’uso delle derivate
  5. Calcolo differenziale
    1. Derivate
    2. Calcolo delle derivate
    3. Retta tangente nel calcolo differenziale
    4. Punti di non derivabilità nel calcolo differenziale
    5. Esercizi sul teorema di Weierstrass con l’uso delle derivate
    6. Studio di funzione completo nel calcolo differenziale
    7. Esercizi teorici nel calcolo differenziale
    8. Metodo di bisezione
    9. Metodo di Newton
  6. Teoremi del calcolo differenziale
    1. Teoria sui Teoremi del calcolo differenziale
    2. Teorema di Rolle
    3. Teorema di Lagrange
    4. Teorema di Cauchy
    5. Teorema di De L’Hôpital
  7. Calcolo integrale
    1. Integrale di Riemann
    2. Integrali immediati
    3. Integrale di funzione composta
    4. Integrali per sostituzione
    5. Integrali per parti
    6. Integrali di funzione razionale
    7. Calcolo delle aree
    8. Metodo dei rettangoli e dei trapezi
    9. Esercizi Misti Integrali Indefiniti
    10. Esercizi Misti Integrali Definiti
  8. Integrali impropri
    1. Teoria Integrali impropri
    2. Carattere di un integrale improprio
    3. Calcolo di un integrale improprio
  9. Espansione di Taylor
    1. Teoria Espansione di Taylor
    2. Limiti di funzione con Taylor
    3. Limiti di successione con Taylor
    4. Stime del resto
  10. Funzioni integrali (Approfondimento)
    1. Teoria Funzioni integrali (Approfondimento)
    2. Studio di funzione integrale
    3. Limiti con Taylor e De L’Hôpital
    4. Derivazione di integrali parametrici (Tecnica di Feynmann)
  11. Numeri Complessi
    1. Teoria Numeri complessi
    2. Espressioni con i numeri complessi
    3. Radice di un numero complesso
    4. Equazioni con i numeri complessi
    5. Disequazioni con i numeri complessi
    6. Esercizi misti Numeri complessi
  12. Serie numeriche
    1. Teoria Serie numeriche
    2. Esercizi Serie a termini positivi
    3. Esercizi Serie a termini di segno variabile
    4. Esercizi Serie geometriche e telescopiche
  13. Successioni di funzioni
    1. Teoria Successioni di funzioni
    2. Esercizi Successioni di funzioni
  14. Serie di funzioni
    1. Teoria Serie di funzioni
    2. Esercizi Serie di funzioni
  15. Serie di potenze
    1. Teoria Serie di potenze
    2. Esercizi Serie di potenze
  16. Serie di Fourier
    1. Teoria Serie di Fourier
    2. Esercizi Serie di Fourier
  17. Trasformata di Fourier
    1. Teoria Trasformata di Fourier
    2. Esercizi Trasformata di Fourier
  18. Funzioni di più variabili
    1. Teoria Funzioni di più variabili
    2. Massimi e minimi liberi e vincolati
    3. Limiti in due variabili
    4. Integrali doppi
    5. Integrali tripli
    6. Integrali di linea di prima specie
    7. Integrali di linea di seconda specie
    8. Forme differenziali e campi vettoriali
    9. Teorema di Gauss-Green
    10. Integrali di superficie
    11. Flusso di un campo vettoriale
    12. Teorema di Stokes
    13. Teorema della divergenza
    14. Campi solenoidali
    15. Teorema del Dini
  19. Equazioni differenziali lineari e non lineari
    1. Teoria equazioni differenziali lineari e non lineari
    2. Equazioni differenziali lineari e non lineari del primo ordine omogenee
  20. Equazioni differenziali lineari
    1. Del primo ordine non omogenee
    2. Di ordine superiore al primo,a coefficienti costanti,omogenee
    3. Di ordine superiore al primo,a coefficienti costanti,non omogenee
    4. Di Eulero,di Bernoulli,di Clairaut,di Lagrange e di Abel
    5. Non omogenee avente per omogenea associata un’equazione di Eulero
    6. Sistemi di EDO
  21. Equazioni differenziali non lineari
    1. A variabili separabiliO
    2. A secondo membro omogeneo
    3. Del tipo y’=y(ax+by+c)
    4. Del tipo y’=y(ax+by+c)/(a’x+b’y+c’)
    5. Equazioni differenziali esatte
    6. Mancanti delle variabili x e y
    7. Cenni sullo studio di un’assegnata equazione differenziale non lineare
    8. Di Riccati
    9. Cambi di variabile: simmetrie di Lie
  22. Analisi complessa
    1. Fondamenti
    2. Funzioni olomorfe
    3. Integrale di Cauchy e applicazioni
    4. Teorema della curva di Jordan e teorema fondamentale dell’Algebra
    5. Teorema di inversione di Lagrange
    6. Teorema dei Residui
    7. Funzioni meromorfe
    8. Prodotti infiniti e prodotti di Weierstrass
    9. Continuazione analitica e topologia
    10. Teoremi di rigidità di funzioni olomorfe
    11. Trasformata di Mellin
  23. Equazioni alle derivate parziali
    1. Equazioni del primo ordine
    2. Equazioni del secondo ordine lineari
    3. Equazioni non-lineari
    4. Sistemi di PDE
  24. Funzioni speciali
    1. Funzione Gamma di Eulero
    2. Funzioni Beta,Digamma,Trigamma
    3. Integrali ellittici
    4. Funzioni di Bessel
    5. Funzione zeta di Riemann e funzioni L di Dirichlet
    6. Funzione polilogaritmo
    7. Funzioni ipergeometriche
  25. Analisi funzionale
    1. Misura e integrale di Lebesgue
    2. Spazi Lp,teoremi di completezza e compattezza
    3. Spazi di Hilbert,serie e trasformata di Fourier
    4. Teoria e pratica dei polinomi ortogonali
    5. Spazi di Sobolev
  26. Complementi
    1. Curiosità e approfondimenti
    2. Compiti di analisi
    3. Esercizi avanzati analisi
  27. Funzioni Convesse

 
 

Tutti gli esercizi di geometria

In questa sezione vengono raccolti molti altri esercizi che coprono tutti gli argomenti di geometria proposti all’interno del sito con lo scopo di offrire al lettore la possibilità di approfondire e rinforzare le proprie competenze inerenti a tali argomenti.

Strutture algebriche.





 
 

Risorse didattiche aggiuntive per approfondire la matematica

Leggi...

  • Math Stack Exchange – Parte della rete Stack Exchange, questo sito è un forum di domande e risposte specificamente dedicato alla matematica. È una delle piattaforme più popolari per discutere e risolvere problemi matematici di vario livello, dall’elementare all’avanzato.
  • Art of Problem Solving (AoPS) – Questo sito è molto noto tra gli studenti di matematica di livello avanzato e i partecipanti a competizioni matematiche. Offre forum, corsi online, e risorse educative su una vasta gamma di argomenti.
  • MathOverflow – Questo sito è destinato a matematici professionisti e ricercatori. È una piattaforma per domande di ricerca avanzata in matematica. È strettamente legato a Math Stack Exchange ma è orientato a un pubblico con una formazione più avanzata.
  • PlanetMath – Una comunità collaborativa di matematici che crea e cura articoli enciclopedici e altre risorse di matematica. È simile a Wikipedia, ma focalizzata esclusivamente sulla matematica.
  • Wolfram MathWorld – Una delle risorse online più complete per la matematica. Contiene migliaia di articoli su argomenti di matematica, creati e curati da esperti. Sebbene non sia un forum, è una risorsa eccellente per la teoria matematica.
  • The Math Forum – Un sito storico che offre un’ampia gamma di risorse, inclusi forum di discussione, articoli e risorse educative. Sebbene alcune parti del sito siano state integrate con altri servizi, come NCTM, rimane una risorsa preziosa per la comunità educativa.
  • Stack Overflow (sezione matematica) – Sebbene Stack Overflow sia principalmente noto per la programmazione, ci sono anche discussioni rilevanti di matematica applicata, specialmente nel contesto della scienza dei dati, statistica, e algoritmi.
  • Reddit (r/Math) – Un subreddit popolare dove si possono trovare discussioni su una vasta gamma di argomenti matematici. È meno formale rispetto ai siti di domande e risposte come Math Stack Exchange, ma ha una comunità attiva e molte discussioni interessanti.
  • Brilliant.org – Offre corsi interattivi e problemi di matematica e scienza. È particolarmente utile per chi vuole allenare le proprie capacità di problem solving in matematica.
  • Khan Academy – Una risorsa educativa globale con lezioni video, esercizi interattivi e articoli su una vasta gamma di argomenti di matematica, dalla scuola elementare all’università.






Document









Document